SlideShare uma empresa Scribd logo
1 de 110
DIABETES MELLITUS
 A 56-year-old woman comes to your clinic for her
annual physical exam. She reports increased
urinary frequency and thirst but is otherwise feeling
generally well. She is obese, does not exercise,
and regularly eats fried foods. A random blood
glucose level is 223 ml/dL, and her hemoglobin A1c
is 9.2.
DEFINITION/ETIOLOGY
 Diabetes mellitus (DM) is defined as persistently
high fasting glucose levels
 greater than 125 on at least 2 separate occasions
 Type 1 DM
 • Onset in childhood
 • Insulin dependent from an early age
 • Not related to obesity
 • Defined as insulin deficiency
 Type 2 DM
 • Onset in adulthood
 • Directly related to obesity
 • Defined as insulin resistance
PRESENTATION
 Polyuria, polyphagia, and polydipsia are the most
common presentation.
 Type 1 diabetics are generally thinner than Type 2
diabetics. Type 2 DM is more resistant to diabetic
ketoacidosis (DKA). Both types present with
 decreased wound healing. Type 2 diabetics are
much less likely to present with polyphagia.
DIAGNOSTIC TESTS
Diabetes is defined/diagnosed as:
 Two fasting blood glucose measurements greater
than 125 mg/dL
 Single glucose level above 200 mg/dL with above
symptoms
 Increased glucose level on oral glucose tolerance
testing
 Hemoglobin A1c >6.5% is a diagnostic criterion and
is the best test to follow response to therapy over
the last several months.
TREATMENT
 Diet, Exercise, and Weight Loss
Weight loss can control as much as 25% of cases of Type 2
DM without the need for medications, since decreasing the
amount of adipose tissue helps to decrease insulin
resistance. Exercising muscle does not need insulin.
 Oral Hypoglycemic Medication
-The best initial drug therapy is with oral metformin.
-Sulfonylureas are not used as first-line therapy because
they increase insulin release from the pancreas,thereby
driving the glucose intracellularly and increasing obesity.
-The goal of therapy is HgA1c <7%.
-Metformin works by blocking gluconeogenesis.
-Thiazoladinediones (glitazones) provide no clear
benefit over the other hypoglycemic medications. They are
relatively contraindicated in CHF because they increase
fluid overload
 Metformln is contrainidicated in those with renal dysfunction
because it can accumulate and cause metabolic acidosis.
 Metformin does not cause hypoglycemia. It is the safest drug
to start in newly diagnosed diabetics.
 Nateglinide and repaglinide are stimulators of insulin release
in a similar manner to sulfonylureas, but do not contain sulfa.
They do not add any therapeutic benefit to sulfonylureas.
 Incretins (exenatide, sitagliptin, saxagliptin,
linagliptin) are part of the mechanism by which oral
glucose normally produces a rise in insulin and
decreases glucagon levels. These agents also
decrease gastric motility and help in weightloss,
decreasing Type 2 diabetes. Exenatide may cause
pancreatitis.
 Alpha glucosidase inhibitors (acarbose,
miglitol) are agents that block glucose absorption
in the bowel. They add about half a point decrease
in HgA1c.They cause flatus, diarrhea, and
abdominal pain. They can be used with renal
insufficiency.
 Pramlintide is an analog of a protein called amylin
that is secreted normally with insulin. Amylin
decreases gastric emptying, decreases glucagon
levels, and decreases appetite.
 Insulin is added if the patient is not controlled with
oral hypoglycemic agents.
 Insulin glargine gives a steady state of insulin for
the entire day. Dosing is not tested. Glargine
provides much more steady blood levels than NPH
insulin,which is dosed twice a day. Long-acting
insulin is combined with a shortacting insulin such
as lispro, aspart, or glulisine.
 Regular insulin is sometimes used as the short-
acting insulin. The goal of therapy is HgA1c <7%.
DIABETIC KETOACIDOSIS(DKA)
 Result of severe insulin deficiency.
 Although more common in those with Type 1 diabetes,
diabetic ketoacidosis
 (DKA) can definitely present in those with Type 2
diabetes.
 Precipitating factor: interrupted insulin deficiency,
infection,emotional stress, excessive alcohol ingestion.
DKA PRESENTATION
• Hyperventilation(kussmaul breathing)
• Possibly altered mental status(coma)
• Metabolic acidosis with an increased anion gap
• Hyperkalemia in blood, but decreased total body
potassium because of urinary spillage
• Serum is positive for ketones
• Nonspecific abdominal pain
• "Acetone" odor on breath
• Polydipsia, polyuria
•Dehydration
•Abdominal pain
•fatal rhythm disturbance
DIAGNOSIS
 Elevated blood glucose level glucose 300 to 800
mg/dL (rarely .1,000),
 Increase serum levels of acetoacetate
 Increase anion gap(sodium-(HCO3+Cl).
 decreased Na,
 normal or Increased serum K1 (total body K1 is
decreased),
 decreased phosphate,
 high anion gap metabolic acidosis,
 serum and urine ketones
DKA
TREATMENT
 Treat with large-volume saline and insulin
replacement. Replace potassium when the
potassium level comes down to a level approaching
normal.
 Correct the underlying cause: noncompliance with
medications, infection,pregnancy, or any serious
illness.
DKA-SUMMARY
a. Extremely low insulin and glucagon excess cause degradation of triglycerides
into fatty acids and eventual conversion into ketoacids.
b. Occurs in patients with DM type I who do not take prescribed insulin or those who have
infections, high stress, myocardial infarction (MI), or high alcohol use
c. H/P = weakness, polyuria, polydipsia, abdominal pain, vomiting; dry mucous
membranes, decreased skin turgor, fruity odor on breath, hyperventilation
(Kussmaul respirations 5 deep, labored, regular breathing); mental status
changes develop with worsening dehydration
d. Labs =
e. Treatment =intravenous (IV) fluids, insulin, KCl; treat underlying disorder
A 57-year-old man is admitted to the intensive care
unit with altered mental status, hyperventilation, and a
markedly elevated glucose level.
Which of the following is the most accurate measure
of the severity of his condition?
a. Glucose level
b. Serum bicarbonate
c. Urine ketones
d. Blood ketones
e. pH level on blood gas
Answer: B. Hyperglycemia is not the best measure of
the severity of DKA. The glucose level can be
markedly elevated without the presence of
ketoacidosis.
Urine ketones mean very little. Although blood
ketones are important, they are not all detected. If the
serum bicarbonate is very low, the patient is at risk of
death. If the serum bicarbonate is high, it does not
matter how high the glucose level is, in terms of
severity. Serum bicarbonate level is a way of saying
"anion gap." If the bicarbonate level is low, the anion
gap is increased.
 A 19-year-old male presents to the emergency room with
altered mental status. History is remarkable for increased
urination over the past few months. On physical
examination, he is a thin, young man with labored
breathing, abdominal tenderness, and mild flank pain.
Temperature is 37.0 degrees Celsius. An arterial blood gas
shows serum pH 7.05, pCO2 17, HCO3 6, pO2 90. This
patient is most likely suffering from which of the following?
 1. Alcohol poisoning
 2. Hormone deficiency
 3. Pyelonephritis
 4. Aortic aneurysm
 5. Ruptured appendix
 PREFERRED RESPONSE ▼ 2
 DISCUSSION: The patient's presentation is consistent with
diabetic ketoacidosis (DKA). DKA is a complication of diabetes
mellitus type I, a deficiency of insulin (a hormone).
 DKA is a medical emergency that occurs in both type I and type
II diabetics, although it is more common in type I. In DKA,
insulin deficiency and glucagon excess promote hyperglycemia
and ketogenesis. Anion gap metabolic acidosis, volume
depletion, and dehydration occur. Left untreated, the condition
may progress to coma and death.
 Trachtenbarg reviews the pathophysiology and treatment of
DKA. Diagnosis is dependent upon specific clinical features,
including glucose greater than 250 mg/dL, pH less than 7.30,
and bicarbonate level less than 18 mEq/L. IV insulin and IV
fluids remain the mainstays of therapy with close monitoring of
potassium levels during the early resuscitation period.
 Incorrect Answers:
Answer 1: Alcohol (EtOH) may cause ketoacidosis
with normal glucose. levels. The presence of
increased urination for months makes DKA more
likely.
Answer 3: Pyelonephritis would present with fever
and severe flank pain. The increased urination is
more consistent with a history of diabetes, making
DKA more likely.
Answer 4: Aortic aneurysms are more common in
older men with a history of smoking. They may
present symptomatically or asymptomatically.
Answer 5: A ruptured appendix presents with
peritoneal signs on physical exam.
 An 18-year-old male with a past medical history of type
I diabetes presents to the emergency room with
polyuria, polydipsia, and dehydration. Vital signs reveal
tachycardia and hypotension. Physical exam is
significant for dry mucous membranes and decreased
skin turgor. In the waiting room he begins vomiting and
complains of intense abdominal pain. You observe him
taking rapid, deep breaths, and over the course of his
brief stay, getting more somnolent. Which of the
following abnormalities would be expected in this
patient?
 1. Hypernatremia
 2. Decreased total body potassium
 3. Hypoglycemia
 4. Absence of urinary beta-OH-butyrate
 5. Non anion-gap metabolic acidosis
 PREFERRED RESPONSE ▼ 2
 This type I diabetic is presenting with signs and symptoms of diabetic
ketoacidosis (DKA). In DKA, total body potassium stores are
generally decreased due to osmotic diuresis.
 DKA is a life-threatening emergency that may occur in either type I or
type II diabetics but is significantly more common in patients with type
I. The pathogenesis is related to insulin deficiency resulting in
hyperglycemia that leads to osmotic diuresis and hypovolemia. The
inability of the body to use the available glucose for ATP production
results in ketone formation and eventually an anion gap metabolic
acidosis. Serum potassium levels may be low, normal, or elevated,
but total body stores are generally low and require repletion. Common
precipitating factors include infection, trauma, myocardial infarction,
sepsis and, of course, inadequate insulin administration. Patients may
present with nausea, vomiting, abdominal pain, Kussmaul respirations
(rapid, deep breaths), dehydration, polydipsia, polyuria and may
eventually progress to altered mental status.
 Diagnosis requires glucose greater than 250 mg/dL, pH
less than 7.3, and bicarbonate less than 18 mEq/L.
Beta-hydroxybutyrate is a better measurement of
ketosis than serum ketones. Regarding the mechanism
of potassium dysregulation, Trachtenbarg explains that
multiple mechanisms are at work. With osmotic diuresis
due to hyperglycemia, much of the serum potassium is
lost. Further, acidosis increases serum potassium levels
and insulin administration lowers them. Thus, significant
shifts occur in the disorder. Once adequate urine output
is confirms and the potassium level is less than 5mEq/L,
potassium administration should be started due to the
usual depletion of total body stores.
 Incorrect Answers:
 Answer 1: Patients with DKA more often present with
hyponatremia. Remember that serum sodium decreases
1.6 mEq/L for every 100 mg/dL increase in glucose.
 Answer 3: DKA requires hyperglycemia by definition.
Hypoglycemia can be a complication of treatment if
glucose is not monitored closely.
 Answer 4: Ketones, such as Beta-OH-Butyrate, are
commonly found in patients with DKA since ketogenesis
is a normal response to starvation caused by inadequate
transit of serum glucose into cells.
 Answer 5: Patients with DKA present with an anion-gap
metabolic acidosis secondary to ketoacids.
HEALTH MAINTENANCE
All patients with DM should receive:
• Pneumococcal vaccine
• Yearly eye exam to check for proliferative
retinopathy, which needs laser therapy
• Statin medication if the LDL is above 100 mg/dL
• ACE inhibitors or ARBs if the blood pressure is
greater than 130/80 mm Hg
• ACEi or ARB if urine tests positive for
microalbuminuria
• Aspirin, used regularly in all diabetic patients above
the age of 30
• Foot exam for neuropathy and ulcers
COMPLICATIONS OF DIABETES
Cardiovascular Complications
 Diabetic patients are at significantly increased risk
of myocardial infarction,stroke, and CHF from
premature atherosclerotic disease.
 This is why the goal of blood pressure in these
patients (below 130/80 mmHg) is lower than in the
general population.
 diabetes is considered an equivalent of coronary
disease for treatment of LDL, and the goal is less
than 100 mg/dL when initiating treatment with
statins.
GASTROPARESIS
 After several years, DM decreases the ability of the
gut to sense the stretch of the walls of the bowel.
Stretch is the main stimulant to gastric motility.
 Gastroparesis is an immobility of the bowels that
leads to bloating, constipation,early satiety,
vomiting, and abdominal discomfort.
 Treatment is with metoclopromide and
erythromycin, which increase gastric motility.
A 53-year-old gentleman presents to your office with a
two-month history of abdominal pain. Of note, the
patient is a long-time patient of yours that you have
been treating for uncontrolled diabetes. In the office
today, his blood sugar is 322 mg/dL. Otherwise, the
patient appears non-toxic and his vital signs are
stable. Upon further questioning, the patient endorses
daily nausea, occasional vomiting, and a feeling of
post-prandial fullness. What is the most appropriate
next step in the patient's care?
1. Radioisotope gastric-emptying scan
2. Upper endoscopy
3. Treatment with metaclopromide; follow up in 3
months
4. Barium radiographic study
5. Hospital admission, nasogastric tube, no PO intake,
fluid resuscitation, and anti-emetics
 PREFERRED RESPONSE ▼ 2
 DISCUSSION: The patient is experiencing diabetic
gastroparesis. The initial work-up includes ruling out
mechanical obstruction via an upper endoscopy study.
 Diabetic gastroparesis is a pathologic state resulting in
delayed gastric emptying. It can be due to injury to the
vagus nerve leading to delayed muscular contraction of
the stomach and ultimately delayed emptying. Diagnosis
is made on the basis of a gastric emptying scan. Prior to
making the diagnosis, ruling out mechanical obstruction
with an upper endoscopy must be undertaken.
 Incorrect Answers:
 Answer 1: Radioisotope gastric-emptying scan is the
test of choice for diagnosing diabetic gastroparesis.
Prior to this study though, it is important to rule out
any obstructing mass with an upper endoscopy.
 Answer 3: Treatment with metaclopromide is the
perferred treatment for diabetic gastroparesis. It is
important however, to confirm the diagnosis and rule
out other pathology.
 Answer 4: Barium radiographs have been used in
the diagnosis of diabetic gastroparesis. However, it
is important to rule out an obstructing mass prior to
obtaining this test.
 Answer 5: This patient does not need admission to
the hospital.
A 57-year-old female presents to her primary care physician
with complaints of nausea, vomiting, abdominal pain, and
bloating that have increased in severity over the past
several months. She reports that she occasionally vomits
after eating; the emesis contains undigested food particles.
Additionally, the patient states that she often is satiated
after only a few bites of food at meals. Her medical history
is significant for hypertension and type II diabetes mellitus
that was first diagnosed 10 years ago. Gastric emptying
scintigraphy is conducted and shows gastric retention of
80% at 2 hours and 40% at 4 hours. Which of the following
is the best first step in management of this patient's
condition. Topic Review Topic
1. Dietary modification
2. Metoclopramide
3. Erythromycin
4. Botulinum toxin injection into pylorus
5. Total parenteral nutrition
 PREFERRED RESPONSE ▼ 1
 DISCUSSION: This patient is suffering from gastroparesis/delayed
gastric emptying. The initial treatment for this condition is diet
modification to frequent, small meals that are low in fiber and fat.
 Presenting symptoms of gastroparesis include nausea, vomiting,
early satiety, postprandial fullness, as well as abdominal pain and
bloating. Management of this condition should first include:
stopping medications that exacerbate gastric stasis, improving
blood glucose control, increasing liquids in the patient's diet,
transitioning to smaller more frequent meals throughout the day,
stopping the use of tobacco and alcohol, as well as decreasing the
amount of insoluble fiber and fat in the patient's diet. If these
preliminary medication treatment options fail, pharmacologic
management can include prokinetic agents such as
metoclopramide and erythromycin.
 The diagnosis of diabetic gastroparesis is made when other
potential causes are excluded and postprandial gastroparesis is
confirmed by a gastric emptying scintigraphy study.
 Incorrect Answers:
 Answers 2,3: Metoclopramide and erythromycin are pro-
kinetic agents that are indicated for the treatment of
gastroparesis; however, dietary modification should be
attempted first before pursuing pharmacologic treatment.
 Answer 4: A botox injection may expedite gastric emptying
and therefore alleviate some of the symptoms of
gastroparesis; however, non-pharmacologic measures
and prokinetic medications should both be attempted
before considering this management option.
 Answer 5: TPN is only required in severe cases of
gastroparesis, in which a patient is not able to obtain
sufficient nutrition; this patient's symptoms are not severe
nor frequent enough to warrant this nutritional support at
this time.
 Illustration B is a gastric emptying scintigraphy study with normal results, as
evidenced by the rapid emptying of the stomach and appearance of tracer in
the small bowel by 2 hours. Illustration C is a gastric emptying scintigraphy
study in a patient with gastroparesis; note how the tracer does not
disappear from the stomach and very little tracer appears distally in the small
intestine.
Illustration B
RETINOPATHY
 DM's effect on microvasculature is especially
apparent in the eye. In the United States, nearly
25,000 people go blind from DM each year.
 The only management for non proliferative
retinopathy is tighter control of glucose. Aspirin
does not help retinopathy.
 When neovascularization and vitreous
hemorrhages are present, it is called proliferative
retinopathy. This is treated with laser
photocoagulation,which markedly retards the
progression to blindness.
DIABETIC NEPHROPATHY
 Diabetes leads to microalbuminuria early in the
disease. The dipstick for urine becomes trace
positive at 300 mg of protein per 24 hours.
 Microalbuminuria means levels of albumin between
30 and 300 mg per 24 hours.
 Patients with DM should be screened annually for
microalbuminuria and started on an ACE inhibitor
or ARB when it is present.
 These agents are proven to decrease the rate of
progression of nephropathy by decreasing
intraglomerular hypertension and decreasing
damage to the kidney.
DIABETIC NEPHROPATHY
a. Intercapillary glomerulosclerosis, mesangial expansion,
and basement membrane degeneration that develops after
long-term DM
b. Slightly greater risk in DM type I than in DM type II
c. Initially presents with proteinuria; renal insufficiency later
develops with Nephrotic syndrome
d. H/P = develops after several years with DM (201); lab
abnormalities may Appear well before symptoms; symptoms
and signs of renal insufficiency
(e.g., HTN, uremia) develop as renal function deteriorates
e. Labs = hypoalbuminemia, increased creatinine
(Cr), increased blood urea nitrogen (BUN);
urinalysis shows proteinuria and microalbuminuria;
electron
microscopy shows basement membrane thickening
and Kimmelstiel-Wilson nodules in glomeruli
DIABETIC NEPHROPATHY
f. Treatment =
 control diabetes;
 angiotensin-converting enzyme inhibitor (ACE-I) or
angiotensin receptor blocker (ARB) to decrease
blood pressure,
 low-protein diet,
 infection prevention;
 dialysis may eventually be required
g. Complications = end-stage renal disease
 A 32-year-old male with type 1 diabetes presents to his
primary care physician concerned about the long-term
consequences of high blood sugar. You discuss the effects
of his condition on the major organ systems. Specifically
relating to the kidneys, which of the following is the earliest
renal abnormality seen in diabetic patients?
 1. Azotemia
 2. Potassium hypofiltration
 3. Na+/H2O retention
 4. Secondary hyperparathyroidism
 5. Glomerular hyperfiltration
 PREFERRED RESPONSE ▼ 5
 DISCUSSION: The earliest renal abnormality seen in
diabetic patients is glomerular hyperfiltration.
 Histologically, high blood glucose results in a thickening of
the glomerular basement membrane followed by
mesangial expansion (Kimmelstiel-Wilson nodules).
These changes alter the filtration unit in a manner that
increases glomerular filtration. This abnormality is the
basis of microalbuminuria screening in all diabetics. As
renal injury progresses, it is possible to quantify the level
of renal injury by the level of albuminuria. In addition to
renal damage, other diabetic vascular complications
include arteriosclerosis leading to hypertension, stroke,
and cardiovascular disease.
 A 45-year-old male with a 15-year history of diabetes
mellitus presents to his primary care provider for a
routine checkup. His doctor is concerned about his renal
function and would like to order a test to detect renal
impairment. Which of the following is the most sensitive
test for detecting renal impairment in diabetic patients?
 1. Cystatin C levels
 2. Urine microalbumin to creatinine ratio
 3. Hemoglobin A1C
 4. Urine protein dipstick
 5. Urinalysis
 PREFERRED RESPONSE ▼ 2
 DISCUSSION: The most sensitive test for detecting renal
impairment in diabetic patients is the urine microalbumin to
creatinine ratio.
 Nephropathy is a microvascular complication of diabetes
mellitus. Diabetic patients undergo thickening of the
glomerular basement membrane and mesangial expansion,
leading to increased glomerular filtration. This results in
microalbuminuria. Patients should be screened for
microalbuminuria with a urine microalbumin to creatinine
ratio in order to detect early renal dysfunction. Patients with
a urine dipstick that is positive for protein have already
progressed to macroalbuminuria.
 diabetes is a major cause of chronic kidney disease. While it
usually takes 10 years from diagnosis to develop in type 1
diabetics, type 2 diabetics may present with nephropathy at
their initial diagnosis of diabetes. Diabetics should be
screened for microalbuminuria with a spot urine
albumin/creatinine ratio. A diagnosis of diabetic nephropathy
can be made if two out of three spot urine albumin/creatinine
ratio tests are positive in a six month period.
 He states that diabetic nephropathy affects approximately 20
to 30% of diabetics. Diabetic nephropathy is associated with
increased cardiovascular mortality. Treatment modalities
known to slow the progression of diabetic nephropathy
include glycemic control, blood pressure control, and ACE
inhibitors.
 Incorrect Answers:
 Answer 1: Cystatin C is a biomarker of kidney function that
is elevated in chronic kidney disease, but it is not the most
sensitive screening test of diabetic nephropathy.
 Answer 3: Hemoglobin A1C levels are used for detecting
the three month average plasma glucose concentration, but
it is not the most sensitive screening test of diabetic
nephropathy.
 Answer 4: Urine protein dipstick can detect
macroalbuminuria, rather than microalbuminuria, and is a
less sensitive test for detecting diabetic nephropathy.
 Answer 5: Urinalysis may be altered in patients with renal
dysfunction, but it is not the most sensitive screening test of
diabetic nephropathy.
A 62-year-old woman presents to the urgent care clinic
complaining of vision changes, headaches, and leg pain
while walking that is relieved by rest, which has been
progressing over many years. She cannot remember the
last time she visited the doctor and she takes no
medications. On physical exam, she is found to have a
blood pressure of 175/95. Her basic metabolic panel is as
follows: Na 132 K 3.8 Cl 102 HCO3 23 BUN 70 Cr 4.2 Glu
360 The patient is diagnosed with end-stage renal failure
and started on dialysis. Screening for which of the following
would have been most appropriate to detect this patient's
progressing renal failure?
1. Hematuria
2. Leukocyturia
3. Urine nitrites
4. Leukocyte esterase
5. Microalbuminuria

PREFERRED RESPONSE ▼ 5
 DISCUSSION: The patient's presentation is consistent with
end-stage renal failure secondary to diabetic nephropathy.
Screening for microalbuminuria (protein) is indicated in patients
with diabetes to detect the development of diabetic
nephropathy.
Important complications of diabetes mellitus include
retinopathy, nephropathy, neuropathy, increased
atherosclerosis, and infections. Diabetic nephropathy occurs in
part due to nonenzymatic glycosylation of the glomerular
basement membrane and the afferent and efferent arterioles,
along with osmotic damage to the glomerular capillary
endothelial cells. Patients with diabetes should be screened for
the presence of small amounts of albumin in the urine
(microalbuminuria) with a spot urine collection. Note that a
urine dipstick is not sensitive enough to detect
microalbuminuria.
 incorrect Answers:
 Answer 1: Hematuria may occur in diabetic
nephropathy, but detection of microalbuminuria is
the most important screening test.
 Answer 2, 3, 4: Leukocyturia, urine nitrites, and
leukocyte esterase are seen in urinary tract
infections
A 65-year-old man with a history of diabetes mellitus type II
presents to his primary care physician for routine care. His
only medication is metformin. His vital signs are:
Temperature: 37.1 Pulse: 80 Blood Pressure: 150/95
Respiratory Rate: 16 SaO2: 99% He is found to be excreting
albumin in his urine at a rate of 150 mg per 24 hours,
compared with 50 mg per 24 hours 3 months ago. What is
the most appropriate next treatment in the care of this
patient?
1. Watchful waiting
2. Decreased protein intake
3. Lisinopril
4. Glyburide
5. Insulin
 PREFERRED RESPONSE ▼ 3
 CORRECT
 DISCUSSION: The patient's history of diabetes along with
hypertension and proteinuria are consistent with diabetic
nephropathy. Blood pressure control, usually with an ACE inhibitor
such as lisinopril, or with a beta-blocker such as atenolol, is the
most appropriate treatment to halt the progression of diabetic
nephropathy.
 The goal of treatment and prevention of diabetic nephropathy is to
lower blood pressure. Control of hypertension is usually achieved
using ACE inhibitors, which provide additional cardiovascular and
mortality reduction benefits. Beta-blockers, although they may
mask the symptoms of hypoglycemia, may be used as second line
agents. In any case, it is important to recognize that the ultimate
desired endpoint is a lower BP.
 Incorrect Answers:
 Answer 1: The patient is a diabetic with
hypertension and requires blood pressure
medication, not observation alone.
 Answer 2: A low protein diet has not been shown to
prevent the progression of diabetic nephropathy.
See Robertson et al. (linked)
 Answer 4 and 5: Insulin and glyburide provide
glycemic control. Glycemic control is not the most
appropriate treatment for the prevention of the
progression of diabetic nephropathy.
DIABETIC NEUROPATHY
a. Neural damage and conduction defects leading to sensory,
motor, and autonomic nerve dysfunction
b. Sensory neuropathy begins in feet and progresses in
stocking-glove pattern;symptoms include paresthesias, neural
pain, and decreased vibratory and pain sensation.
c. Motor neuropathy may be distally or proximally distributed
and may be characterized by weakness or loss of coordination.
d. Autonomic neuropathy can cause postural hypotension,
impotence, incontinence,and diabetic gastroparesis (i.e.,
delayed gastric emptying).
e. Treatment =control diabetes; neural pain can be
treated with tricyclic antidepressants,
carbamazepine, or gabapentin; narcotics or tramadol
can be considered for persistent neural pain; patients
should be taught how to perform
regular foot examinations
f. Complications =Charcot joints, diabetic foot ulcers;
amputation may be needed to treat progressive
infections and deformity
 A 62-year-old female with a history of type II diabetes
presents to her primary care physician for an annual check-
up. Her long-term medications include glyburide, metoprolol,
and sertraline. She is afebrile. Blood pressure is 140/90
mmHg, pulse is 82/min, and respiratory rate is 16/min.
Fasting glucose is recorded as 160 mg/dL. Serum cholesterol
is 150 mg/dL and serum creatinine is 0.9 mg/dL. BMI is 31
kg/m^2. On physical exam, erythema is present at the fifth
metatarsal-phalangeal joint of the right foot and the patient
has decreased sensation over the affected area. Which of the
following would most likely decrease the incidence of future
neuropathy in this patient?
 1. Add atorvastatin
 2. Add hydrochlorothiazide
 3. Right-sided femoral-popliteal bypass
 4. Tight glycemic control
 5. Discontinue sertraline
 PREFERRED RESPONSE ▼ 4
 DISCUSSION: Diabetic neuropathy is a microvascular complication
of diabetes that can contribute to diabetic foot ulcers. Tight glycemic
control is critical for preventing microvascular complications of
diabetes including diabetic neuropathy and retinopathy.
 Elevated glucose levels can affect the osmotic gradient across
neurons and retinal cells through several mechanisms, leading to
cell swelling and ultimately damage. The American Diabetes
Association recommends a goal hemoglobin A1C target of < 7% to
reduce microvascular complications of diabetes in adults. The
elderly and others at risk of hypoglycemia should have a target
hemoglobin A1C of < 8%.
 Bader discusses diabetic foot ulcers: “All patients should have blood
glucose and A1C levels measured at initial presentation and then at
regular intervals. Frequent home blood glucose monitoring is
strongly encouraged. Appropriate therapeutic adjustments (e.g.,
adding or changing oral antihyperglycemic agents, initiating or
increasing insulin) must be made to optimize glycemic control.”
 Incorrect Answers:
 Answer 1: Atorvastatin will help decrease the patient’s
serum cholesterol, which will help reduce the patient’s
risk of macrovascular diabetic complications.
 Answer 2: Hydrochlorothiazide may help decrease this
patient’s blood pressure through fluid diuresis but is
unlikely to improve glycemic control. Hydrochlorothiazide
may raise serum glucose in some individuals.
 Answer 3: Right-sided femoral popliteal bypass is not
indicated at this time, as little evidence is present to
suggest peripheral vascular disease. Furthermore, the
question stem asks about neuropathy, not the condition
of the patient's microvasculature.
 Answer 5: Sertraline is an antidepressant and is not
known to play a role in the formation of diabetic foot
ulcers.
 A 57-year-old with a 30-year-history of type I diabetes
presents to general medical clinic with a lesion on his foot
(Figure A). Although he was advised to wear orthotics by his
podiatrist, he decided to keep wearing his dress shoes and
reports that he observed this lesion when his sock was
stained with blood yesterday morning. His vital signs are
temperature of 37 degrees Celsius, heart rate 75/minute,
blood pressure 145/90 mmHg, respiratory rate 12/minute, and
oxygen saturation 99% on room air. Physical examination
reveals a diminished response to the monofilament test. What
is the most significant risk factor for development of this
condition?
 1. Peripheral vascular disease
 2. Female sex
 3. Duration of diabetes mellitus
 4. Smoking
 5. Neuropath
 PREFERRED RESPONSE ▼ 5
 DISCUSSION: The greatest risk factor for diabetic foot
ulcers is neuropathy. Other risk factors include peripheral
vascular disease, bone abnormalities of the foot, male
sex, smoking, long term diabetes, and a history of a
previous foot ulcer or amputation.
 Diabetic foot is caused by a combination of vascular
disease (ischemia) and neuropathy which lead to ulcers
and infections, sometimes requiring amputation.
Neuropathy is the most important risk factor because
patients do not feel pain and as a result, repetitive injuries
go unnoticed. Furthermore, neuropathy is also a barrier to
healing since patients unable to feel pain are less likely to
offload pressure on the lesion. Management of diabetic
foot ulcers requires offloading with appropriate footware,
debridement, and antibiotic therapy if osteomyelitis or
cellulitis is present. Optimal blood glucose control, and at
times, revascularization also play a role.
 A 58-year-old gentleman comes in to his primary care
physician's office complaining of "tingling in my fingers and
toes". He states that this has occurred more frequently over
the past 3-4 weeks and figured it was about time to see his
doctor. On physical examination he is noted to be obese and
denies any attempts to exercise. A random blood glucose is
found to be 223 mg/dL and his hemoglobin A1c is 9.2. What
is the most likely diagnosis?
 1. Autonomic neuropathy
 2. Compression mononeuropathy
 3. Symmetrical distal polyneuropathy
 4. Proximal neuropathy
 5. Focal neuropathy
 PREFERRED RESPONSE ▼ 3
 DISCUSSION: This individual most likely has diabetes and is
experiencing symmetrical distal polyneuropathy.
 Diabetics can often be affected by peripheral neuropathy,
which is clinically seen as numbness, paresthesias, and pain.
The symptoms of distal or peripheral neuropathy often start
bilaterally in the toes and feet and gradually rise up the
calves and into the knees - the stocking glove" pattern. It's
also common for these individuals to experience a burning
sensation, decreased deep tendon reflexes and vibration
sense along with numbness and paresthesias.
 Incorrect Answers:
 Answer 1: Although autonomic neuropathy can be seen in
diabetics, its presentation is slightly different as it affects the nerves
of internal organs such as the bladder muscles, the cardiovascular
system, the digestive tract, and the genital organs.
 Answer 2: Compressive mononeuropathy refers to a type of
entrapment neuropathy that is caused by peripheral nerve injuries
at specific locations where a nerve is mechanically constricted in a
fibrous or fibro-osseous tunnel.
 Answer 4: Proximal neuropathy can also commonly affect type 2
diabetic patients. It predominantly affects the hips, thighs, buttocks,
or legs and typically begins on one side of the body.
 Answer 5: The head, upper body, or legs may be affected by focal
neuropathy, which appears suddenly and is usually painful. Unlike
the other forms of neuropathy, this type of neuropathy usually goes
away in a few weeks or months and leaves no lasting damage.
 A 63-year-old woman with a history of poorly-controlled
diabetes mellitus presents to your office to review labs and
her blood glucose record. Her hemoglobin A1C is 9.4%. In
addition, her blood glucose records demonstrate poor
control with numerous spikes and lows throughout the day.
Of note, it takes > 1 hr after meals for her blood glucose to
rise. She is also complaining of a decreased appetite,
nausea, reflux, and early satiety. What is the most
appropriate treatment for her current symptoms?
 1. Increasing the dose of her short acting insulin
 2. Loperamide
 3. Metoclopramide
 4. Ondansetron
 5. Gabapentin
 PREFERRED RESPONSE ▼ 3
 DISCUSSION: This patient has signs and symptoms of diabetic
gastroparesis. The appropriate therapy is a prokinetic such as
metoclopramide.
 Diabetic gastroparesis is caused by autonomic neuropathy,
which may occur with poorly controlled type 1 or type 2
diabetes mellitus. Common symptoms include nausea,
vomiting, abdominal pain, early satiety, and poor glucose
control. Diagnosis is made on the basis of a gastric emptying
scan. Treatment includes a low fiber and low residue diet as
well as prokinetic like metoclopramide. Metoclopramide is a
dopaminergic receptor antagonist, and a mixed 5-HT3 receptor
agonist/antagonist ans has both anti-emetic and prokinetic
properties. Severe case of gastroparesis may require a
jejunostomy tube.
 Incorrect Answers:
 Answer 1: Increasing this patient's short acting glucose
would not solve her GI symptoms and may precipitate
hypoglycemic episodes due to her delayed gastric
emptying.
 Answer 2: Loperamide would decrease the activity of the
myenteric plexus likely make her GI symptoms worse.
 Answer 4: Odansetron may help this patient's nausea but
will not improve gastric emptying.
 Answer 5: Gabapentin is sometimes helpful for relief of
symptoms of peripheral neuropathy, but is not regularly
used for gastroparesis.
) A 55-year-old male with a medical history significant for
type II diabetes mellitus, hypertension, and benign prostatic
hypertrophy (BPH) presents to your office with worsening
urinary retention. He was started on tamsulosin for his BPH
one year ago, and had immediate improvement of his urinary
symptoms. However, his urinary symptoms returned and
began to worsen after he was started on several new
medications at an annual checkup three months ago. The
medications started at that time included hydrochlorothiazide
for his hypertension, and a combination of amytriptyline and
gabapentin for severe diabetic neuropathy. What is the likely
culprit of this patient's worsening urinary retention? Topic
Review Topic
1. Hydrochlorothiazide
2. Amitriptyline
3. Gabapentin
4. Tachyphylaxis of tamsulosin
5. Progression of his BPH
 PREFERRED RESPONSE ▼ 2
 DISCUSSION: The anticholinergic effects of the tricyclic (TCA)
amitriptyline is the most likely cause of this patient's worsening
urinary retention issues.
 For medical treatment of diabetic neuropathic pain, first line
therapy is generally a TCA, such as amitryptyline (see
illustration A for an algorithm in treating diabetic neuropathic
pain). If these are contraindicated or do not work alone for the
patient, anticonvulsants and/or serotonin-norepinephrine
reuptake inhibitors (SNRIs) can be used.
 As Lindsay et al. note, tricyclics should not generally be used
in patients over the age of 60 years. They are also relatively
contraindicated in patients with urinary retention issues/BPH
(due to anticholingeric effects), glaucoma, impaired liver
function, thyroid disease, and certain cardiac conditions (e.g.
arrythmias and orthostatic hypertension).
 Incorrect answers:
 Answer 1: The major side effects of
hydrochlorothiazide are hypokalemia,
hyponatremia, hyperglycemia, and hyperlipidemia.
 Answer 3: The major side effects of gabapentin are
somnolence, dizziness and ataxia.
 Answer 4: There is no evidence of tachyphylaxis to
tamsulosin.
 Answer 5: Although progression of BPH would
worsen urinary retention issues, it would not cause
these problems in a short period of time.
) A 52-year-old man comes to your clinic for his annual
physical exam. He is overweight (BMI=31) and is
currently on captopril for hypertension. He reports
increased urinary frequency and thirst but is otherwise
feeling generally well. He does not exercise and
describes his diet as consisting mainly of fast or frozen
food. Which of the following values would be sufficient
to diagnosis this patient with diabetes? Topic Review
Topic
1. Random blood glucose of 185 mg/dL
2. 2 separate fasting glucoses of 130 and 120 mg/dL
3. 2 hour postprandial glucose (glucose tolerance test)
of 180 mg/dL
4. Hemoglobin A1c (HbA1c) of 6.8%
5. The presence of diabetic symptoms only
 PREFERRED RESPONSE ▼ 4
 DISCUSSION: This patient most likely has type II diabetes
mellitus (DM) as a rseult of his high BMI, unhealthy
lifestyle, and increased urinary frequency and thirst. DM
can be diagnosed by a single hemoglobin A1c (HbA1C) of
>6.5%.
 Type 2 diabetes results from insulin insensitivity in
peripheral organs, which results in an increased insulin
requirement. Eventually, the pancreas cannot produce
enough insulin. The condition has a gradual onset and is
typically seen in older individuals, athough it is
increasingly found in children with obesity. The HbA1c
correlates with the mean glucose from the previous 8-12
weeks (approximate lifespan of a red blood cell). The
currently recommended goal for management is < 7%.
 Incorrect Answers:
 Answer 1: DM is diagnosed with a random blood glucose of
at > 200mg/dL AND diabetic symptoms.
 Answer 2: DM is diagnosed with 2 separate fasting glucoses
>126 mg/dL.
 Answer 3: DM is diagnosed with a 2 hour postprandial
glucose (glucose tolerance test) > 200 mg/dL.
 Answer 5: DM diagnosis requires evidence of high glucose
in some form, not simply diabetic symptoms.
HYPEROSMOLAR HYPERGLYCEMIC STATE
(HHS)/HYPEROSMOLAR NONKETOTIC
COMA(HONK)
A. Extremely high glucose with profound dehydration
b. Occurs in patients with DM type II with lengthy
infections, stress, or illness; insulin production is
sufficient to prevent DKA by suppressing lipolysis and
ketogenesis
HHS/HONK SYMPTOMS:
 polyuria,
 polydipsia,
 dehydration,
 mental status changes(convulsion,coma)
 seizures and
 stroke can occur in severe cases
HHS/ HONK DIAGNOSIS& TREATMENT
 glucose >800 mg/dL (frequently .1,000), no acidosis
and extremely high serum osmolality.
 A high BUN(prerenal azotemia) and mild metabolic
acidosis(bicarbonate~20mEq/L) is also seen
without ketosis.
Treatment
 IV fluids,
 insulin,
 correction of electrolyte abnormalities;
 treat underlying disorder
An 81-year-old man with a long history of type II
diabetes presents with altered mental status. He is
accompanied with his wife who reports that he has
become confused over the past twelve hours and has
been complaining of thirst. She also notes that he has
hardly been urinating. Initial assessment in the ER
reveals blood pressure of 90/62 and heart rate of 124
but no fever. The patient is lethargic but has no other
pertinent findings on his physical examination. Initial
labs reveal a serum glucose of 720 mg/dL. What
would be an additional cardinal feature of this clinical
syndrome?
1. Ketosis
2. Severe non-anion gap metabolic acidosis
3. Severe anion gap metabolic acidosis
4. Serum osmolarity > 320 mOsm/L
5. Mortality rate < 1%
 PREFERRED RESPONSE ▼ 4
 This type II diabetic presents with signs and symptoms of
hyperglycemic hyperosmolar nonketotic coma. A feature of this
condition is serum osmolarity > 320 mOsm/L.
 Several different names have been used to describe this state
including hyperosmolar hyperglycemic nonketotic syndrome
(HHNS), hyperosmolar nonketotic coma (HHNC), and
hyperosmolar nonacidotic diabetes. The clinical syndrome refers
to a state of severe hyperglycemia, hyperosmolarity, and
dehydration, most commonly affecting an elderly type II diabetic.
In contrast, type I diabetics commonly present with diabetic
ketoacidosis. Nevertheless, pathogenesis is similar. In HHNS,
low insulin levels cause hyperglycemia resulting in osmotic
diuresis which leads to dehydration. However, ketogenesis is
minimal because a small amount of endogenous insulin is
present, thus preventing significant acidosis. Diagnostic criteria
include hyperglycemia with serum glucose greater than 600
mg/dL and hyperosmolarity with serum osmolarity > 320
mOsm/L, along with no acidosis. Treatment includes fluid and
electrolyte replacement and insulin.
Incorrect Answers:
Answer 1: Ketosis is a laboratory finding in DKA not
HHNS.
Answers 2 and 3: Typically, HHNS presents without
metabolic acidosis. In a minority of cases, it may
present with a mild gap lactic acidosis. However, this
is not a cardinal symptom. In contrast, in DKA, there
is usually a profound anion gap metabolic acidosis
from ketone production.
Answer 5: Mortality rate is 5-10% in DKA but 10-20%
in HHNS. Thus, mortality rate < 1% is not a feature of
either of these illnesses.
 A 61-year-old female with congestive heart failure and
type 2 diabetes is brought to the emergency room by her
husband because of an altered mental status. He states
he normally helps her be compliant with her medications,
but he had been away for several days. On physical
exam, her temperature is 37.2 C, BP 85/55, and HR 130.
Serum glucose is 500 mg/dL. Which of the following is the
first step in the management of this patient? Topic Review
Topic
 1. IV ½ NS
 2. IV NS
 3. IV D5W
 4. Subcutaneous insulin injection
 5. IV insulin
 PREFERRED RESPONSE ▼ 2
 DISCUSSION: The patient in this vignette is most likely
suffering from non-ketotic hyperosmolar syndrome
(NKHS). The first step in the management of this
condition is the administration of IV NS.
 NKHS is a complication of type 2 diabetes. The
concentration of insulin required to prevent ketoacidosis
is lower than the concentration required to adequately
control glucose levels. Thus, the blood glucose may be
very high, but ketoacidosis will be absent (unlike in type
1 DM). NKHS is seen most commonly in older patients
with concurrent cardiac and renal pathology. This
conditon presents with symptoms and signs of shock.
There is a high mortality with NKHS, approximately 50%.
HYPOGLYCEMIA
PRESENTATION
 faintness,
 weakness,
 diaphoresis, and
 palpitations because of responsive excess
secretion of epinephrine (attempt to mobilize
glycogen);
 headache,
 confusion,
 mental status changes, and
 decreased consciousness because of inadequate
supply of glucose to the brain
 A 74-year-old male is brought to the emergency
department by his son during an episode of dizziness,
diaphoresis, and nervousness. The patient's past medical
history includes diabetes mellitus type 2, hypertension,
and end-stage renal disease (ESRD) for which the patient
is on dialysis. Medications include glyburide and
valsartan. Serum glucose is 33 mg/dL, and subsequent
testing shows elevated serum sulfonylurea levels.
Symptoms improve upon intravenous administration of
50% dextrose but blood glucose remains below 80 mg/dL
for 24 hours. Which of the following tests is NOT indicated
in the work-up of this patient's hypoglycemia? Topic
Review Topic
 1. Serum proinsulin level
 2. Urine 5-hydroxyindolacetic acid (5-HIAA) level
 3. Serum insulin-like growth factor II (IGF-II) level
 4. Serum C-peptide level
 5. Serum insulin level
 PREFERRED RESPONSE ▼ 2
 DISCUSSION: The patient presents with dizziness,
diaphoresis, and nervousness and a blood sugar of 33 mg/dL
consistent with symptomatic hypoglycemia.
 Hypoglycemia is associated with multiple differential diagnoses.
In this patient with chronic kidney disease, the use of glyburide,
a sulfonylurea cleared by the kidneys, is the agent most likely
to be responsible for the patient's hypoglycemic episode.
Serum insulin, pro-insulin, c-peptide, and IGF-II levels can help
elucidate alternative diagnoses. Urine 5-HIAA levels are
elevated in carcinoid syndrome and do not contribute to the
work-up of hypoglycemia.
 Havas discusses hypoglycemia in the management of type I
diabetes. Fatigue, mental slowing, confusion, irritability,
weakness, pallor and twitching are all symptoms that may
indicate hypoglycemia. Monitoring blood glucose frequently,
maintaining a prudent diet and regular exercise can help
minimize the risk of significant hypoglycemia.
 Incorrect Answers:
 Answers 1 and 4: Serum C-peptide levels and
proinsulin levels are decreased in the setting of
exogenous insulin use.
 Answer 3: Elevated IGF-II levels can be found in
the setting of acromegaly, a potential underlying
cause of hypoglycemia.
 Answer 5: Serum insulin levels help measure if the
body's response to hypoglycemia is appropriate.
When blood glucose levels are below 60 mg/dL,
serum insulin should be almost entirely suppresed.
 A 52-year-old woman comes to your clinic for her
annual physical exam. She is obese, does not
exercise, and regularly eats fried foods. A random
blood glucose is 249 mg/dL. Her hemoglobin A1C is
9.5. Which of the following treatments would be
weight neutral or cause weight loss in this patient?
Topic Review Topic
 1. Glargine
 2. Glyburide
 3. Pioglitazone
 4. Metformin
 5. Glipizide
 4
 DISCUSSION: The patient in this vignette most likely has
type II diabetes. Of the given treatments, only metformin is
weight neutral in the majority of cases (i.e. it does not
cause significant weight gain/loss).
 Metformin is a first-line treatment for type II DM in most
patients. Although the exact mechanism is unknown, it
appears to decrease gluconeogenesis and increase insulin
sensitivity. There is no risk of hypoglycemia or weight gain
(though some patients even lose weight). The most high
yield side effect involves lactic acidosis, particularly in
patients with renal insufficiency.
 incorrect Answers:
 Answer 1: Glargine, a long-acting insulin, can
cause weight gain.
 Answer 2: Glyburide, a second generation
sulfonylurea, can cause weight gain.
 Answer 3: Pioglitazone, a thiazolidinedione, can
cause weight gain.
 Answer 5: Glipizide, a second generation
sulfonylurea, can cause weight gain.
Diabetes mellitus

Mais conteúdo relacionado

Mais procurados

Diabetes mellitus with complication
Diabetes mellitus with complicationDiabetes mellitus with complication
Diabetes mellitus with complicationDipali Dumbre
 
Diabetes mellitus
Diabetes mellitusDiabetes mellitus
Diabetes mellitusAbhra Ghosh
 
Diagnosis of diabetes mellitus
Diagnosis of diabetes mellitus  Diagnosis of diabetes mellitus
Diagnosis of diabetes mellitus Dilek Gogas Yavuz
 
Pediatrics diabetic mellitus
Pediatrics diabetic mellitusPediatrics diabetic mellitus
Pediatrics diabetic mellitusaklilu abrham
 
...DIAGNOSIS OF DIABETES MELLITUS...
...DIAGNOSIS OF DIABETES MELLITUS......DIAGNOSIS OF DIABETES MELLITUS...
...DIAGNOSIS OF DIABETES MELLITUS...Dr.Subir Kumar
 
Diabetes diagnosis
Diabetes diagnosis Diabetes diagnosis
Diabetes diagnosis ikramdr01
 
Diabetes Diagnosis and Classification
Diabetes Diagnosis and ClassificationDiabetes Diagnosis and Classification
Diabetes Diagnosis and ClassificationDR. VIVEK ARYA
 
C:\documents and settings\administrator\桌面\35 ndiabetes mellitus
C:\documents and settings\administrator\桌面\35 ndiabetes mellitusC:\documents and settings\administrator\桌面\35 ndiabetes mellitus
C:\documents and settings\administrator\桌面\35 ndiabetes mellitusinternalmed
 
Hypoglycemia by Dr Shubham Jain
Hypoglycemia by Dr Shubham JainHypoglycemia by Dr Shubham Jain
Hypoglycemia by Dr Shubham JainShubham Jain
 
Diabetes mellitus
Diabetes mellitusDiabetes mellitus
Diabetes mellitusSara Ravi
 

Mais procurados (20)

Diabetes mellitus with complication
Diabetes mellitus with complicationDiabetes mellitus with complication
Diabetes mellitus with complication
 
Pathophysiology and Classification of diabetes by Dr Selim
Pathophysiology and Classification of diabetes by Dr SelimPathophysiology and Classification of diabetes by Dr Selim
Pathophysiology and Classification of diabetes by Dr Selim
 
Complications of diabetes
Complications of diabetes Complications of diabetes
Complications of diabetes
 
Diabetes treatment
Diabetes treatmentDiabetes treatment
Diabetes treatment
 
Diabetes mellitus
Diabetes mellitusDiabetes mellitus
Diabetes mellitus
 
Diagnosis of diabetes mellitus
Diagnosis of diabetes mellitus  Diagnosis of diabetes mellitus
Diagnosis of diabetes mellitus
 
Pediatrics diabetic mellitus
Pediatrics diabetic mellitusPediatrics diabetic mellitus
Pediatrics diabetic mellitus
 
...DIAGNOSIS OF DIABETES MELLITUS...
...DIAGNOSIS OF DIABETES MELLITUS......DIAGNOSIS OF DIABETES MELLITUS...
...DIAGNOSIS OF DIABETES MELLITUS...
 
Diabetes diagnosis
Diabetes diagnosis Diabetes diagnosis
Diabetes diagnosis
 
Diabetes mellitus type 2
Diabetes mellitus type 2Diabetes mellitus type 2
Diabetes mellitus type 2
 
2. diabetes mellitus
2. diabetes mellitus2. diabetes mellitus
2. diabetes mellitus
 
Type 2 dm
Type 2 dmType 2 dm
Type 2 dm
 
Diabetes Diagnosis and Classification
Diabetes Diagnosis and ClassificationDiabetes Diagnosis and Classification
Diabetes Diagnosis and Classification
 
C:\documents and settings\administrator\桌面\35 ndiabetes mellitus
C:\documents and settings\administrator\桌面\35 ndiabetes mellitusC:\documents and settings\administrator\桌面\35 ndiabetes mellitus
C:\documents and settings\administrator\桌面\35 ndiabetes mellitus
 
Hypoglycemia in dm patients
Hypoglycemia in dm patientsHypoglycemia in dm patients
Hypoglycemia in dm patients
 
hypoglycemia
hypoglycemiahypoglycemia
hypoglycemia
 
Hypoglycemia by Dr Shubham Jain
Hypoglycemia by Dr Shubham JainHypoglycemia by Dr Shubham Jain
Hypoglycemia by Dr Shubham Jain
 
Diabetes Mellitus
Diabetes MellitusDiabetes Mellitus
Diabetes Mellitus
 
Diabetes mellitus
Diabetes mellitusDiabetes mellitus
Diabetes mellitus
 
Hypoglycemia- Assessment and Treatment
Hypoglycemia- Assessment and TreatmentHypoglycemia- Assessment and Treatment
Hypoglycemia- Assessment and Treatment
 

Semelhante a Diabetes mellitus

Diabetes paediatrics
Diabetes paediatricsDiabetes paediatrics
Diabetes paediatricsGeorge Muli
 
General Medicine -Diabetes slides.pptx
General Medicine -Diabetes slides.pptxGeneral Medicine -Diabetes slides.pptx
General Medicine -Diabetes slides.pptxANDREWODHIAMBO12
 
Hypoglycemia among diabetic patients, 11 Dec 2021.pptx
Hypoglycemia among diabetic patients, 11 Dec 2021.pptxHypoglycemia among diabetic patients, 11 Dec 2021.pptx
Hypoglycemia among diabetic patients, 11 Dec 2021.pptxAhmed Elshebiny
 
Hypoglycemia
HypoglycemiaHypoglycemia
Hypoglycemiayuyuricci
 
3. Management of patients with diabetes.pptx
3. Management of patients with diabetes.pptx3. Management of patients with diabetes.pptx
3. Management of patients with diabetes.pptxDrChandiniRavikumar
 
Pathophysiology of diabetes mellitus
Pathophysiology of diabetes mellitusPathophysiology of diabetes mellitus
Pathophysiology of diabetes mellitusniva niva
 
Current management of Diabetes Mellitus by Ghaza khan.
Current management of Diabetes Mellitus by Ghaza khan.Current management of Diabetes Mellitus by Ghaza khan.
Current management of Diabetes Mellitus by Ghaza khan.Ghaza Khan
 
Current Management of Diabetes Mellitus by Ghaza khan.
Current Management of Diabetes Mellitus by Ghaza khan.Current Management of Diabetes Mellitus by Ghaza khan.
Current Management of Diabetes Mellitus by Ghaza khan.Ghaza Khan
 
diabetes mellitus om verma.pdf
diabetes mellitus om verma.pdfdiabetes mellitus om verma.pdf
diabetes mellitus om verma.pdfOM VERMA
 
Hypoglycaemia Biochemistry decrease in Glucose mechanism
 Hypoglycaemia Biochemistry decrease in Glucose mechanism Hypoglycaemia Biochemistry decrease in Glucose mechanism
Hypoglycaemia Biochemistry decrease in Glucose mechanismMirzaNaadir
 
diabetesmellitus.pptx
diabetesmellitus.pptxdiabetesmellitus.pptx
diabetesmellitus.pptxDishaThakur53
 
Metabolic-Emergencies.pptx
Metabolic-Emergencies.pptxMetabolic-Emergencies.pptx
Metabolic-Emergencies.pptxWengelRedkiss
 
A practice to diabetes guidelines
A practice to diabetes guidelinesA practice to diabetes guidelines
A practice to diabetes guidelinesWalid Babikr
 
Diabetic emergencies
Diabetic emergenciesDiabetic emergencies
Diabetic emergenciesRichardKeshi
 
Diabetes mellitus acute complications
Diabetes mellitus   acute complicationsDiabetes mellitus   acute complications
Diabetes mellitus acute complicationsswapru
 
Determination of Blood Glucose Using Glusose Oxidase-Peroxidase Method
Determination of Blood Glucose Using Glusose Oxidase-Peroxidase MethodDetermination of Blood Glucose Using Glusose Oxidase-Peroxidase Method
Determination of Blood Glucose Using Glusose Oxidase-Peroxidase MethodZoldylck
 

Semelhante a Diabetes mellitus (20)

Diabetes paediatrics
Diabetes paediatricsDiabetes paediatrics
Diabetes paediatrics
 
General Medicine -Diabetes slides.pptx
General Medicine -Diabetes slides.pptxGeneral Medicine -Diabetes slides.pptx
General Medicine -Diabetes slides.pptx
 
Hypoglycemia among diabetic patients, 11 Dec 2021.pptx
Hypoglycemia among diabetic patients, 11 Dec 2021.pptxHypoglycemia among diabetic patients, 11 Dec 2021.pptx
Hypoglycemia among diabetic patients, 11 Dec 2021.pptx
 
Hypoglycemia
HypoglycemiaHypoglycemia
Hypoglycemia
 
Anaesth. consideration endocrine 2
Anaesth. consideration endocrine 2Anaesth. consideration endocrine 2
Anaesth. consideration endocrine 2
 
3. Management of patients with diabetes.pptx
3. Management of patients with diabetes.pptx3. Management of patients with diabetes.pptx
3. Management of patients with diabetes.pptx
 
Diabetes Mellitus Type 2 - Pathology.pptx
Diabetes Mellitus Type 2 - Pathology.pptxDiabetes Mellitus Type 2 - Pathology.pptx
Diabetes Mellitus Type 2 - Pathology.pptx
 
Pathophysiology of diabetes mellitus
Pathophysiology of diabetes mellitusPathophysiology of diabetes mellitus
Pathophysiology of diabetes mellitus
 
Current management of Diabetes Mellitus by Ghaza khan.
Current management of Diabetes Mellitus by Ghaza khan.Current management of Diabetes Mellitus by Ghaza khan.
Current management of Diabetes Mellitus by Ghaza khan.
 
Current Management of Diabetes Mellitus by Ghaza khan.
Current Management of Diabetes Mellitus by Ghaza khan.Current Management of Diabetes Mellitus by Ghaza khan.
Current Management of Diabetes Mellitus by Ghaza khan.
 
diabetes mellitus om verma.pdf
diabetes mellitus om verma.pdfdiabetes mellitus om verma.pdf
diabetes mellitus om verma.pdf
 
Hypoglycaemia Biochemistry decrease in Glucose mechanism
 Hypoglycaemia Biochemistry decrease in Glucose mechanism Hypoglycaemia Biochemistry decrease in Glucose mechanism
Hypoglycaemia Biochemistry decrease in Glucose mechanism
 
diabetes & perio
 diabetes & perio diabetes & perio
diabetes & perio
 
diabetesmellitus.pptx
diabetesmellitus.pptxdiabetesmellitus.pptx
diabetesmellitus.pptx
 
Metabolic-Emergencies.pptx
Metabolic-Emergencies.pptxMetabolic-Emergencies.pptx
Metabolic-Emergencies.pptx
 
A practice to diabetes guidelines
A practice to diabetes guidelinesA practice to diabetes guidelines
A practice to diabetes guidelines
 
Diabetic emergencies
Diabetic emergenciesDiabetic emergencies
Diabetic emergencies
 
Diabetes mellitus acute complications
Diabetes mellitus   acute complicationsDiabetes mellitus   acute complications
Diabetes mellitus acute complications
 
Diabetes
DiabetesDiabetes
Diabetes
 
Determination of Blood Glucose Using Glusose Oxidase-Peroxidase Method
Determination of Blood Glucose Using Glusose Oxidase-Peroxidase MethodDetermination of Blood Glucose Using Glusose Oxidase-Peroxidase Method
Determination of Blood Glucose Using Glusose Oxidase-Peroxidase Method
 

Mais de Lih Yin Chong

Management of abdominal trauma
Management of abdominal traumaManagement of abdominal trauma
Management of abdominal traumaLih Yin Chong
 
Peripheral Nerve block(ankle block,wrist block, digital block)
Peripheral Nerve block(ankle block,wrist block, digital block)Peripheral Nerve block(ankle block,wrist block, digital block)
Peripheral Nerve block(ankle block,wrist block, digital block)Lih Yin Chong
 
peptic ulcer disease
peptic ulcer diseasepeptic ulcer disease
peptic ulcer diseaseLih Yin Chong
 
Skin condition associated with disease
Skin condition associated with diseaseSkin condition associated with disease
Skin condition associated with diseaseLih Yin Chong
 
Evaluating of fetal heart tracing
Evaluating of fetal heart tracingEvaluating of fetal heart tracing
Evaluating of fetal heart tracingLih Yin Chong
 
Congestive heart failure
Congestive heart failureCongestive heart failure
Congestive heart failureLih Yin Chong
 
Cardiac rhythm disorders
Cardiac rhythm disordersCardiac rhythm disorders
Cardiac rhythm disordersLih Yin Chong
 
Coronary artery disease
Coronary artery diseaseCoronary artery disease
Coronary artery diseaseLih Yin Chong
 
Acute coronary syndrome
Acute coronary syndromeAcute coronary syndrome
Acute coronary syndromeLih Yin Chong
 
Disease of adrenal gland
Disease of adrenal glandDisease of adrenal gland
Disease of adrenal glandLih Yin Chong
 
Endocrinology thyroid disorder
Endocrinology thyroid disorderEndocrinology thyroid disorder
Endocrinology thyroid disorderLih Yin Chong
 
Endocrinology parathyroid gland
Endocrinology parathyroid glandEndocrinology parathyroid gland
Endocrinology parathyroid glandLih Yin Chong
 
Endocrinology thyroid disorder
Endocrinology thyroid disorderEndocrinology thyroid disorder
Endocrinology thyroid disorderLih Yin Chong
 
Endocrinology -pituitary gland
Endocrinology -pituitary glandEndocrinology -pituitary gland
Endocrinology -pituitary glandLih Yin Chong
 

Mais de Lih Yin Chong (18)

Copd-2019
Copd-2019Copd-2019
Copd-2019
 
Management of abdominal trauma
Management of abdominal traumaManagement of abdominal trauma
Management of abdominal trauma
 
Peripheral Nerve block(ankle block,wrist block, digital block)
Peripheral Nerve block(ankle block,wrist block, digital block)Peripheral Nerve block(ankle block,wrist block, digital block)
Peripheral Nerve block(ankle block,wrist block, digital block)
 
Esophageal disease
Esophageal diseaseEsophageal disease
Esophageal disease
 
peptic ulcer disease
peptic ulcer diseasepeptic ulcer disease
peptic ulcer disease
 
Skin condition associated with disease
Skin condition associated with diseaseSkin condition associated with disease
Skin condition associated with disease
 
Orthopedics
OrthopedicsOrthopedics
Orthopedics
 
Breast pathology
Breast pathologyBreast pathology
Breast pathology
 
Evaluating of fetal heart tracing
Evaluating of fetal heart tracingEvaluating of fetal heart tracing
Evaluating of fetal heart tracing
 
Congestive heart failure
Congestive heart failureCongestive heart failure
Congestive heart failure
 
Cardiac rhythm disorders
Cardiac rhythm disordersCardiac rhythm disorders
Cardiac rhythm disorders
 
Coronary artery disease
Coronary artery diseaseCoronary artery disease
Coronary artery disease
 
Acute coronary syndrome
Acute coronary syndromeAcute coronary syndrome
Acute coronary syndrome
 
Disease of adrenal gland
Disease of adrenal glandDisease of adrenal gland
Disease of adrenal gland
 
Endocrinology thyroid disorder
Endocrinology thyroid disorderEndocrinology thyroid disorder
Endocrinology thyroid disorder
 
Endocrinology parathyroid gland
Endocrinology parathyroid glandEndocrinology parathyroid gland
Endocrinology parathyroid gland
 
Endocrinology thyroid disorder
Endocrinology thyroid disorderEndocrinology thyroid disorder
Endocrinology thyroid disorder
 
Endocrinology -pituitary gland
Endocrinology -pituitary glandEndocrinology -pituitary gland
Endocrinology -pituitary gland
 

Último

Night 7k to 12k Chennai City Center Call Girls 👉👉 7427069034⭐⭐ 100% Genuine E...
Night 7k to 12k Chennai City Center Call Girls 👉👉 7427069034⭐⭐ 100% Genuine E...Night 7k to 12k Chennai City Center Call Girls 👉👉 7427069034⭐⭐ 100% Genuine E...
Night 7k to 12k Chennai City Center Call Girls 👉👉 7427069034⭐⭐ 100% Genuine E...hotbabesbook
 
VIP Call Girls Indore Kirti 💚😋 9256729539 🚀 Indore Escorts
VIP Call Girls Indore Kirti 💚😋  9256729539 🚀 Indore EscortsVIP Call Girls Indore Kirti 💚😋  9256729539 🚀 Indore Escorts
VIP Call Girls Indore Kirti 💚😋 9256729539 🚀 Indore Escortsaditipandeya
 
VIP Call Girls Tirunelveli Aaradhya 8250192130 Independent Escort Service Tir...
VIP Call Girls Tirunelveli Aaradhya 8250192130 Independent Escort Service Tir...VIP Call Girls Tirunelveli Aaradhya 8250192130 Independent Escort Service Tir...
VIP Call Girls Tirunelveli Aaradhya 8250192130 Independent Escort Service Tir...narwatsonia7
 
Chandrapur Call girls 8617370543 Provides all area service COD available
Chandrapur Call girls 8617370543 Provides all area service COD availableChandrapur Call girls 8617370543 Provides all area service COD available
Chandrapur Call girls 8617370543 Provides all area service COD availableDipal Arora
 
Call Girls Bareilly Just Call 9907093804 Top Class Call Girl Service Available
Call Girls Bareilly Just Call 9907093804 Top Class Call Girl Service AvailableCall Girls Bareilly Just Call 9907093804 Top Class Call Girl Service Available
Call Girls Bareilly Just Call 9907093804 Top Class Call Girl Service AvailableDipal Arora
 
Call Girls Siliguri Just Call 9907093804 Top Class Call Girl Service Available
Call Girls Siliguri Just Call 9907093804 Top Class Call Girl Service AvailableCall Girls Siliguri Just Call 9907093804 Top Class Call Girl Service Available
Call Girls Siliguri Just Call 9907093804 Top Class Call Girl Service AvailableDipal Arora
 
Premium Call Girls Cottonpet Whatsapp 7001035870 Independent Escort Service
Premium Call Girls Cottonpet Whatsapp 7001035870 Independent Escort ServicePremium Call Girls Cottonpet Whatsapp 7001035870 Independent Escort Service
Premium Call Girls Cottonpet Whatsapp 7001035870 Independent Escort Servicevidya singh
 
Top Rated Hyderabad Call Girls Erragadda ⟟ 6297143586 ⟟ Call Me For Genuine ...
Top Rated  Hyderabad Call Girls Erragadda ⟟ 6297143586 ⟟ Call Me For Genuine ...Top Rated  Hyderabad Call Girls Erragadda ⟟ 6297143586 ⟟ Call Me For Genuine ...
Top Rated Hyderabad Call Girls Erragadda ⟟ 6297143586 ⟟ Call Me For Genuine ...chandars293
 
Night 7k to 12k Navi Mumbai Call Girl Photo 👉 BOOK NOW 9833363713 👈 ♀️ night ...
Night 7k to 12k Navi Mumbai Call Girl Photo 👉 BOOK NOW 9833363713 👈 ♀️ night ...Night 7k to 12k Navi Mumbai Call Girl Photo 👉 BOOK NOW 9833363713 👈 ♀️ night ...
Night 7k to 12k Navi Mumbai Call Girl Photo 👉 BOOK NOW 9833363713 👈 ♀️ night ...aartirawatdelhi
 
💎VVIP Kolkata Call Girls Parganas🩱7001035870🩱Independent Girl ( Ac Rooms Avai...
💎VVIP Kolkata Call Girls Parganas🩱7001035870🩱Independent Girl ( Ac Rooms Avai...💎VVIP Kolkata Call Girls Parganas🩱7001035870🩱Independent Girl ( Ac Rooms Avai...
💎VVIP Kolkata Call Girls Parganas🩱7001035870🩱Independent Girl ( Ac Rooms Avai...Taniya Sharma
 
Call Girls Faridabad Just Call 9907093804 Top Class Call Girl Service Available
Call Girls Faridabad Just Call 9907093804 Top Class Call Girl Service AvailableCall Girls Faridabad Just Call 9907093804 Top Class Call Girl Service Available
Call Girls Faridabad Just Call 9907093804 Top Class Call Girl Service AvailableDipal Arora
 
Call Girls Service Surat Samaira ❤️🍑 8250192130 👄 Independent Escort Service ...
Call Girls Service Surat Samaira ❤️🍑 8250192130 👄 Independent Escort Service ...Call Girls Service Surat Samaira ❤️🍑 8250192130 👄 Independent Escort Service ...
Call Girls Service Surat Samaira ❤️🍑 8250192130 👄 Independent Escort Service ...CALL GIRLS
 
Bangalore Call Girl Whatsapp Number 100% Complete Your Sexual Needs
Bangalore Call Girl Whatsapp Number 100% Complete Your Sexual NeedsBangalore Call Girl Whatsapp Number 100% Complete Your Sexual Needs
Bangalore Call Girl Whatsapp Number 100% Complete Your Sexual NeedsGfnyt
 
Russian Call Girls in Jaipur Riya WhatsApp ❤8445551418 VIP Call Girls Jaipur
Russian Call Girls in Jaipur Riya WhatsApp ❤8445551418 VIP Call Girls JaipurRussian Call Girls in Jaipur Riya WhatsApp ❤8445551418 VIP Call Girls Jaipur
Russian Call Girls in Jaipur Riya WhatsApp ❤8445551418 VIP Call Girls Jaipurparulsinha
 
Call Girls Kochi Just Call 9907093804 Top Class Call Girl Service Available
Call Girls Kochi Just Call 9907093804 Top Class Call Girl Service AvailableCall Girls Kochi Just Call 9907093804 Top Class Call Girl Service Available
Call Girls Kochi Just Call 9907093804 Top Class Call Girl Service AvailableDipal Arora
 
Top Rated Bangalore Call Girls Mg Road ⟟ 8250192130 ⟟ Call Me For Genuine Sex...
Top Rated Bangalore Call Girls Mg Road ⟟ 8250192130 ⟟ Call Me For Genuine Sex...Top Rated Bangalore Call Girls Mg Road ⟟ 8250192130 ⟟ Call Me For Genuine Sex...
Top Rated Bangalore Call Girls Mg Road ⟟ 8250192130 ⟟ Call Me For Genuine Sex...narwatsonia7
 
Call Girls Bhubaneswar Just Call 9907093804 Top Class Call Girl Service Avail...
Call Girls Bhubaneswar Just Call 9907093804 Top Class Call Girl Service Avail...Call Girls Bhubaneswar Just Call 9907093804 Top Class Call Girl Service Avail...
Call Girls Bhubaneswar Just Call 9907093804 Top Class Call Girl Service Avail...Dipal Arora
 
Best Rate (Guwahati ) Call Girls Guwahati ⟟ 8617370543 ⟟ High Class Call Girl...
Best Rate (Guwahati ) Call Girls Guwahati ⟟ 8617370543 ⟟ High Class Call Girl...Best Rate (Guwahati ) Call Girls Guwahati ⟟ 8617370543 ⟟ High Class Call Girl...
Best Rate (Guwahati ) Call Girls Guwahati ⟟ 8617370543 ⟟ High Class Call Girl...Dipal Arora
 
Call Girls Service Jaipur Grishma WhatsApp ❤8445551418 VIP Call Girls Jaipur
Call Girls Service Jaipur Grishma WhatsApp ❤8445551418 VIP Call Girls JaipurCall Girls Service Jaipur Grishma WhatsApp ❤8445551418 VIP Call Girls Jaipur
Call Girls Service Jaipur Grishma WhatsApp ❤8445551418 VIP Call Girls Jaipurparulsinha
 

Último (20)

Night 7k to 12k Chennai City Center Call Girls 👉👉 7427069034⭐⭐ 100% Genuine E...
Night 7k to 12k Chennai City Center Call Girls 👉👉 7427069034⭐⭐ 100% Genuine E...Night 7k to 12k Chennai City Center Call Girls 👉👉 7427069034⭐⭐ 100% Genuine E...
Night 7k to 12k Chennai City Center Call Girls 👉👉 7427069034⭐⭐ 100% Genuine E...
 
VIP Call Girls Indore Kirti 💚😋 9256729539 🚀 Indore Escorts
VIP Call Girls Indore Kirti 💚😋  9256729539 🚀 Indore EscortsVIP Call Girls Indore Kirti 💚😋  9256729539 🚀 Indore Escorts
VIP Call Girls Indore Kirti 💚😋 9256729539 🚀 Indore Escorts
 
VIP Call Girls Tirunelveli Aaradhya 8250192130 Independent Escort Service Tir...
VIP Call Girls Tirunelveli Aaradhya 8250192130 Independent Escort Service Tir...VIP Call Girls Tirunelveli Aaradhya 8250192130 Independent Escort Service Tir...
VIP Call Girls Tirunelveli Aaradhya 8250192130 Independent Escort Service Tir...
 
Chandrapur Call girls 8617370543 Provides all area service COD available
Chandrapur Call girls 8617370543 Provides all area service COD availableChandrapur Call girls 8617370543 Provides all area service COD available
Chandrapur Call girls 8617370543 Provides all area service COD available
 
Call Girls Bareilly Just Call 9907093804 Top Class Call Girl Service Available
Call Girls Bareilly Just Call 9907093804 Top Class Call Girl Service AvailableCall Girls Bareilly Just Call 9907093804 Top Class Call Girl Service Available
Call Girls Bareilly Just Call 9907093804 Top Class Call Girl Service Available
 
Call Girls Siliguri Just Call 9907093804 Top Class Call Girl Service Available
Call Girls Siliguri Just Call 9907093804 Top Class Call Girl Service AvailableCall Girls Siliguri Just Call 9907093804 Top Class Call Girl Service Available
Call Girls Siliguri Just Call 9907093804 Top Class Call Girl Service Available
 
Premium Call Girls Cottonpet Whatsapp 7001035870 Independent Escort Service
Premium Call Girls Cottonpet Whatsapp 7001035870 Independent Escort ServicePremium Call Girls Cottonpet Whatsapp 7001035870 Independent Escort Service
Premium Call Girls Cottonpet Whatsapp 7001035870 Independent Escort Service
 
Top Rated Hyderabad Call Girls Erragadda ⟟ 6297143586 ⟟ Call Me For Genuine ...
Top Rated  Hyderabad Call Girls Erragadda ⟟ 6297143586 ⟟ Call Me For Genuine ...Top Rated  Hyderabad Call Girls Erragadda ⟟ 6297143586 ⟟ Call Me For Genuine ...
Top Rated Hyderabad Call Girls Erragadda ⟟ 6297143586 ⟟ Call Me For Genuine ...
 
Night 7k to 12k Navi Mumbai Call Girl Photo 👉 BOOK NOW 9833363713 👈 ♀️ night ...
Night 7k to 12k Navi Mumbai Call Girl Photo 👉 BOOK NOW 9833363713 👈 ♀️ night ...Night 7k to 12k Navi Mumbai Call Girl Photo 👉 BOOK NOW 9833363713 👈 ♀️ night ...
Night 7k to 12k Navi Mumbai Call Girl Photo 👉 BOOK NOW 9833363713 👈 ♀️ night ...
 
💎VVIP Kolkata Call Girls Parganas🩱7001035870🩱Independent Girl ( Ac Rooms Avai...
💎VVIP Kolkata Call Girls Parganas🩱7001035870🩱Independent Girl ( Ac Rooms Avai...💎VVIP Kolkata Call Girls Parganas🩱7001035870🩱Independent Girl ( Ac Rooms Avai...
💎VVIP Kolkata Call Girls Parganas🩱7001035870🩱Independent Girl ( Ac Rooms Avai...
 
Call Girls Faridabad Just Call 9907093804 Top Class Call Girl Service Available
Call Girls Faridabad Just Call 9907093804 Top Class Call Girl Service AvailableCall Girls Faridabad Just Call 9907093804 Top Class Call Girl Service Available
Call Girls Faridabad Just Call 9907093804 Top Class Call Girl Service Available
 
Call Girls Service Surat Samaira ❤️🍑 8250192130 👄 Independent Escort Service ...
Call Girls Service Surat Samaira ❤️🍑 8250192130 👄 Independent Escort Service ...Call Girls Service Surat Samaira ❤️🍑 8250192130 👄 Independent Escort Service ...
Call Girls Service Surat Samaira ❤️🍑 8250192130 👄 Independent Escort Service ...
 
Bangalore Call Girl Whatsapp Number 100% Complete Your Sexual Needs
Bangalore Call Girl Whatsapp Number 100% Complete Your Sexual NeedsBangalore Call Girl Whatsapp Number 100% Complete Your Sexual Needs
Bangalore Call Girl Whatsapp Number 100% Complete Your Sexual Needs
 
Russian Call Girls in Jaipur Riya WhatsApp ❤8445551418 VIP Call Girls Jaipur
Russian Call Girls in Jaipur Riya WhatsApp ❤8445551418 VIP Call Girls JaipurRussian Call Girls in Jaipur Riya WhatsApp ❤8445551418 VIP Call Girls Jaipur
Russian Call Girls in Jaipur Riya WhatsApp ❤8445551418 VIP Call Girls Jaipur
 
Call Girls Kochi Just Call 9907093804 Top Class Call Girl Service Available
Call Girls Kochi Just Call 9907093804 Top Class Call Girl Service AvailableCall Girls Kochi Just Call 9907093804 Top Class Call Girl Service Available
Call Girls Kochi Just Call 9907093804 Top Class Call Girl Service Available
 
Top Rated Bangalore Call Girls Mg Road ⟟ 8250192130 ⟟ Call Me For Genuine Sex...
Top Rated Bangalore Call Girls Mg Road ⟟ 8250192130 ⟟ Call Me For Genuine Sex...Top Rated Bangalore Call Girls Mg Road ⟟ 8250192130 ⟟ Call Me For Genuine Sex...
Top Rated Bangalore Call Girls Mg Road ⟟ 8250192130 ⟟ Call Me For Genuine Sex...
 
Call Girls Bhubaneswar Just Call 9907093804 Top Class Call Girl Service Avail...
Call Girls Bhubaneswar Just Call 9907093804 Top Class Call Girl Service Avail...Call Girls Bhubaneswar Just Call 9907093804 Top Class Call Girl Service Avail...
Call Girls Bhubaneswar Just Call 9907093804 Top Class Call Girl Service Avail...
 
Russian Call Girls in Delhi Tanvi ➡️ 9711199012 💋📞 Independent Escort Service...
Russian Call Girls in Delhi Tanvi ➡️ 9711199012 💋📞 Independent Escort Service...Russian Call Girls in Delhi Tanvi ➡️ 9711199012 💋📞 Independent Escort Service...
Russian Call Girls in Delhi Tanvi ➡️ 9711199012 💋📞 Independent Escort Service...
 
Best Rate (Guwahati ) Call Girls Guwahati ⟟ 8617370543 ⟟ High Class Call Girl...
Best Rate (Guwahati ) Call Girls Guwahati ⟟ 8617370543 ⟟ High Class Call Girl...Best Rate (Guwahati ) Call Girls Guwahati ⟟ 8617370543 ⟟ High Class Call Girl...
Best Rate (Guwahati ) Call Girls Guwahati ⟟ 8617370543 ⟟ High Class Call Girl...
 
Call Girls Service Jaipur Grishma WhatsApp ❤8445551418 VIP Call Girls Jaipur
Call Girls Service Jaipur Grishma WhatsApp ❤8445551418 VIP Call Girls JaipurCall Girls Service Jaipur Grishma WhatsApp ❤8445551418 VIP Call Girls Jaipur
Call Girls Service Jaipur Grishma WhatsApp ❤8445551418 VIP Call Girls Jaipur
 

Diabetes mellitus

  • 2.  A 56-year-old woman comes to your clinic for her annual physical exam. She reports increased urinary frequency and thirst but is otherwise feeling generally well. She is obese, does not exercise, and regularly eats fried foods. A random blood glucose level is 223 ml/dL, and her hemoglobin A1c is 9.2.
  • 3. DEFINITION/ETIOLOGY  Diabetes mellitus (DM) is defined as persistently high fasting glucose levels  greater than 125 on at least 2 separate occasions
  • 4.  Type 1 DM  • Onset in childhood  • Insulin dependent from an early age  • Not related to obesity  • Defined as insulin deficiency  Type 2 DM  • Onset in adulthood  • Directly related to obesity  • Defined as insulin resistance
  • 5.
  • 6. PRESENTATION  Polyuria, polyphagia, and polydipsia are the most common presentation.  Type 1 diabetics are generally thinner than Type 2 diabetics. Type 2 DM is more resistant to diabetic ketoacidosis (DKA). Both types present with  decreased wound healing. Type 2 diabetics are much less likely to present with polyphagia.
  • 7. DIAGNOSTIC TESTS Diabetes is defined/diagnosed as:  Two fasting blood glucose measurements greater than 125 mg/dL  Single glucose level above 200 mg/dL with above symptoms  Increased glucose level on oral glucose tolerance testing  Hemoglobin A1c >6.5% is a diagnostic criterion and is the best test to follow response to therapy over the last several months.
  • 8.
  • 9. TREATMENT  Diet, Exercise, and Weight Loss Weight loss can control as much as 25% of cases of Type 2 DM without the need for medications, since decreasing the amount of adipose tissue helps to decrease insulin resistance. Exercising muscle does not need insulin.
  • 10.  Oral Hypoglycemic Medication -The best initial drug therapy is with oral metformin. -Sulfonylureas are not used as first-line therapy because they increase insulin release from the pancreas,thereby driving the glucose intracellularly and increasing obesity. -The goal of therapy is HgA1c <7%. -Metformin works by blocking gluconeogenesis. -Thiazoladinediones (glitazones) provide no clear benefit over the other hypoglycemic medications. They are relatively contraindicated in CHF because they increase fluid overload
  • 11.  Metformln is contrainidicated in those with renal dysfunction because it can accumulate and cause metabolic acidosis.  Metformin does not cause hypoglycemia. It is the safest drug to start in newly diagnosed diabetics.  Nateglinide and repaglinide are stimulators of insulin release in a similar manner to sulfonylureas, but do not contain sulfa. They do not add any therapeutic benefit to sulfonylureas.
  • 12.  Incretins (exenatide, sitagliptin, saxagliptin, linagliptin) are part of the mechanism by which oral glucose normally produces a rise in insulin and decreases glucagon levels. These agents also decrease gastric motility and help in weightloss, decreasing Type 2 diabetes. Exenatide may cause pancreatitis.  Alpha glucosidase inhibitors (acarbose, miglitol) are agents that block glucose absorption in the bowel. They add about half a point decrease in HgA1c.They cause flatus, diarrhea, and abdominal pain. They can be used with renal insufficiency.
  • 13.
  • 14.  Pramlintide is an analog of a protein called amylin that is secreted normally with insulin. Amylin decreases gastric emptying, decreases glucagon levels, and decreases appetite.  Insulin is added if the patient is not controlled with oral hypoglycemic agents.
  • 15.  Insulin glargine gives a steady state of insulin for the entire day. Dosing is not tested. Glargine provides much more steady blood levels than NPH insulin,which is dosed twice a day. Long-acting insulin is combined with a shortacting insulin such as lispro, aspart, or glulisine.  Regular insulin is sometimes used as the short- acting insulin. The goal of therapy is HgA1c <7%.
  • 16.
  • 17.
  • 18. DIABETIC KETOACIDOSIS(DKA)  Result of severe insulin deficiency.  Although more common in those with Type 1 diabetes, diabetic ketoacidosis  (DKA) can definitely present in those with Type 2 diabetes.  Precipitating factor: interrupted insulin deficiency, infection,emotional stress, excessive alcohol ingestion.
  • 19.
  • 20. DKA PRESENTATION • Hyperventilation(kussmaul breathing) • Possibly altered mental status(coma) • Metabolic acidosis with an increased anion gap • Hyperkalemia in blood, but decreased total body potassium because of urinary spillage • Serum is positive for ketones • Nonspecific abdominal pain • "Acetone" odor on breath • Polydipsia, polyuria •Dehydration •Abdominal pain •fatal rhythm disturbance
  • 21. DIAGNOSIS  Elevated blood glucose level glucose 300 to 800 mg/dL (rarely .1,000),  Increase serum levels of acetoacetate  Increase anion gap(sodium-(HCO3+Cl).  decreased Na,  normal or Increased serum K1 (total body K1 is decreased),  decreased phosphate,  high anion gap metabolic acidosis,  serum and urine ketones
  • 22. DKA
  • 23. TREATMENT  Treat with large-volume saline and insulin replacement. Replace potassium when the potassium level comes down to a level approaching normal.  Correct the underlying cause: noncompliance with medications, infection,pregnancy, or any serious illness.
  • 24. DKA-SUMMARY a. Extremely low insulin and glucagon excess cause degradation of triglycerides into fatty acids and eventual conversion into ketoacids. b. Occurs in patients with DM type I who do not take prescribed insulin or those who have infections, high stress, myocardial infarction (MI), or high alcohol use c. H/P = weakness, polyuria, polydipsia, abdominal pain, vomiting; dry mucous membranes, decreased skin turgor, fruity odor on breath, hyperventilation (Kussmaul respirations 5 deep, labored, regular breathing); mental status changes develop with worsening dehydration d. Labs = e. Treatment =intravenous (IV) fluids, insulin, KCl; treat underlying disorder
  • 25. A 57-year-old man is admitted to the intensive care unit with altered mental status, hyperventilation, and a markedly elevated glucose level. Which of the following is the most accurate measure of the severity of his condition? a. Glucose level b. Serum bicarbonate c. Urine ketones d. Blood ketones e. pH level on blood gas
  • 26. Answer: B. Hyperglycemia is not the best measure of the severity of DKA. The glucose level can be markedly elevated without the presence of ketoacidosis. Urine ketones mean very little. Although blood ketones are important, they are not all detected. If the serum bicarbonate is very low, the patient is at risk of death. If the serum bicarbonate is high, it does not matter how high the glucose level is, in terms of severity. Serum bicarbonate level is a way of saying "anion gap." If the bicarbonate level is low, the anion gap is increased.
  • 27.  A 19-year-old male presents to the emergency room with altered mental status. History is remarkable for increased urination over the past few months. On physical examination, he is a thin, young man with labored breathing, abdominal tenderness, and mild flank pain. Temperature is 37.0 degrees Celsius. An arterial blood gas shows serum pH 7.05, pCO2 17, HCO3 6, pO2 90. This patient is most likely suffering from which of the following?  1. Alcohol poisoning  2. Hormone deficiency  3. Pyelonephritis  4. Aortic aneurysm  5. Ruptured appendix
  • 28.  PREFERRED RESPONSE ▼ 2  DISCUSSION: The patient's presentation is consistent with diabetic ketoacidosis (DKA). DKA is a complication of diabetes mellitus type I, a deficiency of insulin (a hormone).  DKA is a medical emergency that occurs in both type I and type II diabetics, although it is more common in type I. In DKA, insulin deficiency and glucagon excess promote hyperglycemia and ketogenesis. Anion gap metabolic acidosis, volume depletion, and dehydration occur. Left untreated, the condition may progress to coma and death.  Trachtenbarg reviews the pathophysiology and treatment of DKA. Diagnosis is dependent upon specific clinical features, including glucose greater than 250 mg/dL, pH less than 7.30, and bicarbonate level less than 18 mEq/L. IV insulin and IV fluids remain the mainstays of therapy with close monitoring of potassium levels during the early resuscitation period.
  • 29.  Incorrect Answers: Answer 1: Alcohol (EtOH) may cause ketoacidosis with normal glucose. levels. The presence of increased urination for months makes DKA more likely. Answer 3: Pyelonephritis would present with fever and severe flank pain. The increased urination is more consistent with a history of diabetes, making DKA more likely. Answer 4: Aortic aneurysms are more common in older men with a history of smoking. They may present symptomatically or asymptomatically. Answer 5: A ruptured appendix presents with peritoneal signs on physical exam.
  • 30.
  • 31.  An 18-year-old male with a past medical history of type I diabetes presents to the emergency room with polyuria, polydipsia, and dehydration. Vital signs reveal tachycardia and hypotension. Physical exam is significant for dry mucous membranes and decreased skin turgor. In the waiting room he begins vomiting and complains of intense abdominal pain. You observe him taking rapid, deep breaths, and over the course of his brief stay, getting more somnolent. Which of the following abnormalities would be expected in this patient?  1. Hypernatremia  2. Decreased total body potassium  3. Hypoglycemia  4. Absence of urinary beta-OH-butyrate  5. Non anion-gap metabolic acidosis
  • 32.  PREFERRED RESPONSE ▼ 2  This type I diabetic is presenting with signs and symptoms of diabetic ketoacidosis (DKA). In DKA, total body potassium stores are generally decreased due to osmotic diuresis.  DKA is a life-threatening emergency that may occur in either type I or type II diabetics but is significantly more common in patients with type I. The pathogenesis is related to insulin deficiency resulting in hyperglycemia that leads to osmotic diuresis and hypovolemia. The inability of the body to use the available glucose for ATP production results in ketone formation and eventually an anion gap metabolic acidosis. Serum potassium levels may be low, normal, or elevated, but total body stores are generally low and require repletion. Common precipitating factors include infection, trauma, myocardial infarction, sepsis and, of course, inadequate insulin administration. Patients may present with nausea, vomiting, abdominal pain, Kussmaul respirations (rapid, deep breaths), dehydration, polydipsia, polyuria and may eventually progress to altered mental status.
  • 33.  Diagnosis requires glucose greater than 250 mg/dL, pH less than 7.3, and bicarbonate less than 18 mEq/L. Beta-hydroxybutyrate is a better measurement of ketosis than serum ketones. Regarding the mechanism of potassium dysregulation, Trachtenbarg explains that multiple mechanisms are at work. With osmotic diuresis due to hyperglycemia, much of the serum potassium is lost. Further, acidosis increases serum potassium levels and insulin administration lowers them. Thus, significant shifts occur in the disorder. Once adequate urine output is confirms and the potassium level is less than 5mEq/L, potassium administration should be started due to the usual depletion of total body stores.
  • 34.  Incorrect Answers:  Answer 1: Patients with DKA more often present with hyponatremia. Remember that serum sodium decreases 1.6 mEq/L for every 100 mg/dL increase in glucose.  Answer 3: DKA requires hyperglycemia by definition. Hypoglycemia can be a complication of treatment if glucose is not monitored closely.  Answer 4: Ketones, such as Beta-OH-Butyrate, are commonly found in patients with DKA since ketogenesis is a normal response to starvation caused by inadequate transit of serum glucose into cells.  Answer 5: Patients with DKA present with an anion-gap metabolic acidosis secondary to ketoacids.
  • 35. HEALTH MAINTENANCE All patients with DM should receive: • Pneumococcal vaccine • Yearly eye exam to check for proliferative retinopathy, which needs laser therapy • Statin medication if the LDL is above 100 mg/dL • ACE inhibitors or ARBs if the blood pressure is greater than 130/80 mm Hg • ACEi or ARB if urine tests positive for microalbuminuria • Aspirin, used regularly in all diabetic patients above the age of 30 • Foot exam for neuropathy and ulcers
  • 36. COMPLICATIONS OF DIABETES Cardiovascular Complications  Diabetic patients are at significantly increased risk of myocardial infarction,stroke, and CHF from premature atherosclerotic disease.  This is why the goal of blood pressure in these patients (below 130/80 mmHg) is lower than in the general population.  diabetes is considered an equivalent of coronary disease for treatment of LDL, and the goal is less than 100 mg/dL when initiating treatment with statins.
  • 37. GASTROPARESIS  After several years, DM decreases the ability of the gut to sense the stretch of the walls of the bowel. Stretch is the main stimulant to gastric motility.  Gastroparesis is an immobility of the bowels that leads to bloating, constipation,early satiety, vomiting, and abdominal discomfort.  Treatment is with metoclopromide and erythromycin, which increase gastric motility.
  • 38.
  • 39.
  • 40.
  • 41. A 53-year-old gentleman presents to your office with a two-month history of abdominal pain. Of note, the patient is a long-time patient of yours that you have been treating for uncontrolled diabetes. In the office today, his blood sugar is 322 mg/dL. Otherwise, the patient appears non-toxic and his vital signs are stable. Upon further questioning, the patient endorses daily nausea, occasional vomiting, and a feeling of post-prandial fullness. What is the most appropriate next step in the patient's care? 1. Radioisotope gastric-emptying scan 2. Upper endoscopy 3. Treatment with metaclopromide; follow up in 3 months 4. Barium radiographic study 5. Hospital admission, nasogastric tube, no PO intake, fluid resuscitation, and anti-emetics
  • 42.  PREFERRED RESPONSE ▼ 2  DISCUSSION: The patient is experiencing diabetic gastroparesis. The initial work-up includes ruling out mechanical obstruction via an upper endoscopy study.  Diabetic gastroparesis is a pathologic state resulting in delayed gastric emptying. It can be due to injury to the vagus nerve leading to delayed muscular contraction of the stomach and ultimately delayed emptying. Diagnosis is made on the basis of a gastric emptying scan. Prior to making the diagnosis, ruling out mechanical obstruction with an upper endoscopy must be undertaken.
  • 43.  Incorrect Answers:  Answer 1: Radioisotope gastric-emptying scan is the test of choice for diagnosing diabetic gastroparesis. Prior to this study though, it is important to rule out any obstructing mass with an upper endoscopy.  Answer 3: Treatment with metaclopromide is the perferred treatment for diabetic gastroparesis. It is important however, to confirm the diagnosis and rule out other pathology.  Answer 4: Barium radiographs have been used in the diagnosis of diabetic gastroparesis. However, it is important to rule out an obstructing mass prior to obtaining this test.  Answer 5: This patient does not need admission to the hospital.
  • 44. A 57-year-old female presents to her primary care physician with complaints of nausea, vomiting, abdominal pain, and bloating that have increased in severity over the past several months. She reports that she occasionally vomits after eating; the emesis contains undigested food particles. Additionally, the patient states that she often is satiated after only a few bites of food at meals. Her medical history is significant for hypertension and type II diabetes mellitus that was first diagnosed 10 years ago. Gastric emptying scintigraphy is conducted and shows gastric retention of 80% at 2 hours and 40% at 4 hours. Which of the following is the best first step in management of this patient's condition. Topic Review Topic 1. Dietary modification 2. Metoclopramide 3. Erythromycin 4. Botulinum toxin injection into pylorus 5. Total parenteral nutrition
  • 45.  PREFERRED RESPONSE ▼ 1  DISCUSSION: This patient is suffering from gastroparesis/delayed gastric emptying. The initial treatment for this condition is diet modification to frequent, small meals that are low in fiber and fat.  Presenting symptoms of gastroparesis include nausea, vomiting, early satiety, postprandial fullness, as well as abdominal pain and bloating. Management of this condition should first include: stopping medications that exacerbate gastric stasis, improving blood glucose control, increasing liquids in the patient's diet, transitioning to smaller more frequent meals throughout the day, stopping the use of tobacco and alcohol, as well as decreasing the amount of insoluble fiber and fat in the patient's diet. If these preliminary medication treatment options fail, pharmacologic management can include prokinetic agents such as metoclopramide and erythromycin.  The diagnosis of diabetic gastroparesis is made when other potential causes are excluded and postprandial gastroparesis is confirmed by a gastric emptying scintigraphy study.
  • 46.  Incorrect Answers:  Answers 2,3: Metoclopramide and erythromycin are pro- kinetic agents that are indicated for the treatment of gastroparesis; however, dietary modification should be attempted first before pursuing pharmacologic treatment.  Answer 4: A botox injection may expedite gastric emptying and therefore alleviate some of the symptoms of gastroparesis; however, non-pharmacologic measures and prokinetic medications should both be attempted before considering this management option.  Answer 5: TPN is only required in severe cases of gastroparesis, in which a patient is not able to obtain sufficient nutrition; this patient's symptoms are not severe nor frequent enough to warrant this nutritional support at this time.
  • 47.
  • 48.  Illustration B is a gastric emptying scintigraphy study with normal results, as evidenced by the rapid emptying of the stomach and appearance of tracer in the small bowel by 2 hours. Illustration C is a gastric emptying scintigraphy study in a patient with gastroparesis; note how the tracer does not disappear from the stomach and very little tracer appears distally in the small intestine. Illustration B
  • 49. RETINOPATHY  DM's effect on microvasculature is especially apparent in the eye. In the United States, nearly 25,000 people go blind from DM each year.  The only management for non proliferative retinopathy is tighter control of glucose. Aspirin does not help retinopathy.  When neovascularization and vitreous hemorrhages are present, it is called proliferative retinopathy. This is treated with laser photocoagulation,which markedly retards the progression to blindness.
  • 50.
  • 51. DIABETIC NEPHROPATHY  Diabetes leads to microalbuminuria early in the disease. The dipstick for urine becomes trace positive at 300 mg of protein per 24 hours.  Microalbuminuria means levels of albumin between 30 and 300 mg per 24 hours.  Patients with DM should be screened annually for microalbuminuria and started on an ACE inhibitor or ARB when it is present.  These agents are proven to decrease the rate of progression of nephropathy by decreasing intraglomerular hypertension and decreasing damage to the kidney.
  • 52. DIABETIC NEPHROPATHY a. Intercapillary glomerulosclerosis, mesangial expansion, and basement membrane degeneration that develops after long-term DM b. Slightly greater risk in DM type I than in DM type II c. Initially presents with proteinuria; renal insufficiency later develops with Nephrotic syndrome d. H/P = develops after several years with DM (201); lab abnormalities may Appear well before symptoms; symptoms and signs of renal insufficiency (e.g., HTN, uremia) develop as renal function deteriorates
  • 53. e. Labs = hypoalbuminemia, increased creatinine (Cr), increased blood urea nitrogen (BUN); urinalysis shows proteinuria and microalbuminuria; electron microscopy shows basement membrane thickening and Kimmelstiel-Wilson nodules in glomeruli
  • 54. DIABETIC NEPHROPATHY f. Treatment =  control diabetes;  angiotensin-converting enzyme inhibitor (ACE-I) or angiotensin receptor blocker (ARB) to decrease blood pressure,  low-protein diet,  infection prevention;  dialysis may eventually be required g. Complications = end-stage renal disease
  • 55.  A 32-year-old male with type 1 diabetes presents to his primary care physician concerned about the long-term consequences of high blood sugar. You discuss the effects of his condition on the major organ systems. Specifically relating to the kidneys, which of the following is the earliest renal abnormality seen in diabetic patients?  1. Azotemia  2. Potassium hypofiltration  3. Na+/H2O retention  4. Secondary hyperparathyroidism  5. Glomerular hyperfiltration
  • 56.  PREFERRED RESPONSE ▼ 5  DISCUSSION: The earliest renal abnormality seen in diabetic patients is glomerular hyperfiltration.  Histologically, high blood glucose results in a thickening of the glomerular basement membrane followed by mesangial expansion (Kimmelstiel-Wilson nodules). These changes alter the filtration unit in a manner that increases glomerular filtration. This abnormality is the basis of microalbuminuria screening in all diabetics. As renal injury progresses, it is possible to quantify the level of renal injury by the level of albuminuria. In addition to renal damage, other diabetic vascular complications include arteriosclerosis leading to hypertension, stroke, and cardiovascular disease.
  • 57.  A 45-year-old male with a 15-year history of diabetes mellitus presents to his primary care provider for a routine checkup. His doctor is concerned about his renal function and would like to order a test to detect renal impairment. Which of the following is the most sensitive test for detecting renal impairment in diabetic patients?  1. Cystatin C levels  2. Urine microalbumin to creatinine ratio  3. Hemoglobin A1C  4. Urine protein dipstick  5. Urinalysis
  • 58.  PREFERRED RESPONSE ▼ 2  DISCUSSION: The most sensitive test for detecting renal impairment in diabetic patients is the urine microalbumin to creatinine ratio.  Nephropathy is a microvascular complication of diabetes mellitus. Diabetic patients undergo thickening of the glomerular basement membrane and mesangial expansion, leading to increased glomerular filtration. This results in microalbuminuria. Patients should be screened for microalbuminuria with a urine microalbumin to creatinine ratio in order to detect early renal dysfunction. Patients with a urine dipstick that is positive for protein have already progressed to macroalbuminuria.
  • 59.  diabetes is a major cause of chronic kidney disease. While it usually takes 10 years from diagnosis to develop in type 1 diabetics, type 2 diabetics may present with nephropathy at their initial diagnosis of diabetes. Diabetics should be screened for microalbuminuria with a spot urine albumin/creatinine ratio. A diagnosis of diabetic nephropathy can be made if two out of three spot urine albumin/creatinine ratio tests are positive in a six month period.  He states that diabetic nephropathy affects approximately 20 to 30% of diabetics. Diabetic nephropathy is associated with increased cardiovascular mortality. Treatment modalities known to slow the progression of diabetic nephropathy include glycemic control, blood pressure control, and ACE inhibitors.
  • 60.  Incorrect Answers:  Answer 1: Cystatin C is a biomarker of kidney function that is elevated in chronic kidney disease, but it is not the most sensitive screening test of diabetic nephropathy.  Answer 3: Hemoglobin A1C levels are used for detecting the three month average plasma glucose concentration, but it is not the most sensitive screening test of diabetic nephropathy.  Answer 4: Urine protein dipstick can detect macroalbuminuria, rather than microalbuminuria, and is a less sensitive test for detecting diabetic nephropathy.  Answer 5: Urinalysis may be altered in patients with renal dysfunction, but it is not the most sensitive screening test of diabetic nephropathy.
  • 61. A 62-year-old woman presents to the urgent care clinic complaining of vision changes, headaches, and leg pain while walking that is relieved by rest, which has been progressing over many years. She cannot remember the last time she visited the doctor and she takes no medications. On physical exam, she is found to have a blood pressure of 175/95. Her basic metabolic panel is as follows: Na 132 K 3.8 Cl 102 HCO3 23 BUN 70 Cr 4.2 Glu 360 The patient is diagnosed with end-stage renal failure and started on dialysis. Screening for which of the following would have been most appropriate to detect this patient's progressing renal failure? 1. Hematuria 2. Leukocyturia 3. Urine nitrites 4. Leukocyte esterase 5. Microalbuminuria
  • 62.  PREFERRED RESPONSE ▼ 5  DISCUSSION: The patient's presentation is consistent with end-stage renal failure secondary to diabetic nephropathy. Screening for microalbuminuria (protein) is indicated in patients with diabetes to detect the development of diabetic nephropathy. Important complications of diabetes mellitus include retinopathy, nephropathy, neuropathy, increased atherosclerosis, and infections. Diabetic nephropathy occurs in part due to nonenzymatic glycosylation of the glomerular basement membrane and the afferent and efferent arterioles, along with osmotic damage to the glomerular capillary endothelial cells. Patients with diabetes should be screened for the presence of small amounts of albumin in the urine (microalbuminuria) with a spot urine collection. Note that a urine dipstick is not sensitive enough to detect microalbuminuria.
  • 63.  incorrect Answers:  Answer 1: Hematuria may occur in diabetic nephropathy, but detection of microalbuminuria is the most important screening test.  Answer 2, 3, 4: Leukocyturia, urine nitrites, and leukocyte esterase are seen in urinary tract infections
  • 64. A 65-year-old man with a history of diabetes mellitus type II presents to his primary care physician for routine care. His only medication is metformin. His vital signs are: Temperature: 37.1 Pulse: 80 Blood Pressure: 150/95 Respiratory Rate: 16 SaO2: 99% He is found to be excreting albumin in his urine at a rate of 150 mg per 24 hours, compared with 50 mg per 24 hours 3 months ago. What is the most appropriate next treatment in the care of this patient? 1. Watchful waiting 2. Decreased protein intake 3. Lisinopril 4. Glyburide 5. Insulin
  • 65.  PREFERRED RESPONSE ▼ 3  CORRECT  DISCUSSION: The patient's history of diabetes along with hypertension and proteinuria are consistent with diabetic nephropathy. Blood pressure control, usually with an ACE inhibitor such as lisinopril, or with a beta-blocker such as atenolol, is the most appropriate treatment to halt the progression of diabetic nephropathy.  The goal of treatment and prevention of diabetic nephropathy is to lower blood pressure. Control of hypertension is usually achieved using ACE inhibitors, which provide additional cardiovascular and mortality reduction benefits. Beta-blockers, although they may mask the symptoms of hypoglycemia, may be used as second line agents. In any case, it is important to recognize that the ultimate desired endpoint is a lower BP.
  • 66.  Incorrect Answers:  Answer 1: The patient is a diabetic with hypertension and requires blood pressure medication, not observation alone.  Answer 2: A low protein diet has not been shown to prevent the progression of diabetic nephropathy. See Robertson et al. (linked)  Answer 4 and 5: Insulin and glyburide provide glycemic control. Glycemic control is not the most appropriate treatment for the prevention of the progression of diabetic nephropathy.
  • 67.
  • 68. DIABETIC NEUROPATHY a. Neural damage and conduction defects leading to sensory, motor, and autonomic nerve dysfunction b. Sensory neuropathy begins in feet and progresses in stocking-glove pattern;symptoms include paresthesias, neural pain, and decreased vibratory and pain sensation. c. Motor neuropathy may be distally or proximally distributed and may be characterized by weakness or loss of coordination. d. Autonomic neuropathy can cause postural hypotension, impotence, incontinence,and diabetic gastroparesis (i.e., delayed gastric emptying).
  • 69. e. Treatment =control diabetes; neural pain can be treated with tricyclic antidepressants, carbamazepine, or gabapentin; narcotics or tramadol can be considered for persistent neural pain; patients should be taught how to perform regular foot examinations f. Complications =Charcot joints, diabetic foot ulcers; amputation may be needed to treat progressive infections and deformity
  • 70.
  • 71.  A 62-year-old female with a history of type II diabetes presents to her primary care physician for an annual check- up. Her long-term medications include glyburide, metoprolol, and sertraline. She is afebrile. Blood pressure is 140/90 mmHg, pulse is 82/min, and respiratory rate is 16/min. Fasting glucose is recorded as 160 mg/dL. Serum cholesterol is 150 mg/dL and serum creatinine is 0.9 mg/dL. BMI is 31 kg/m^2. On physical exam, erythema is present at the fifth metatarsal-phalangeal joint of the right foot and the patient has decreased sensation over the affected area. Which of the following would most likely decrease the incidence of future neuropathy in this patient?  1. Add atorvastatin  2. Add hydrochlorothiazide  3. Right-sided femoral-popliteal bypass  4. Tight glycemic control  5. Discontinue sertraline
  • 72.  PREFERRED RESPONSE ▼ 4  DISCUSSION: Diabetic neuropathy is a microvascular complication of diabetes that can contribute to diabetic foot ulcers. Tight glycemic control is critical for preventing microvascular complications of diabetes including diabetic neuropathy and retinopathy.  Elevated glucose levels can affect the osmotic gradient across neurons and retinal cells through several mechanisms, leading to cell swelling and ultimately damage. The American Diabetes Association recommends a goal hemoglobin A1C target of < 7% to reduce microvascular complications of diabetes in adults. The elderly and others at risk of hypoglycemia should have a target hemoglobin A1C of < 8%.  Bader discusses diabetic foot ulcers: “All patients should have blood glucose and A1C levels measured at initial presentation and then at regular intervals. Frequent home blood glucose monitoring is strongly encouraged. Appropriate therapeutic adjustments (e.g., adding or changing oral antihyperglycemic agents, initiating or increasing insulin) must be made to optimize glycemic control.”
  • 73.  Incorrect Answers:  Answer 1: Atorvastatin will help decrease the patient’s serum cholesterol, which will help reduce the patient’s risk of macrovascular diabetic complications.  Answer 2: Hydrochlorothiazide may help decrease this patient’s blood pressure through fluid diuresis but is unlikely to improve glycemic control. Hydrochlorothiazide may raise serum glucose in some individuals.  Answer 3: Right-sided femoral popliteal bypass is not indicated at this time, as little evidence is present to suggest peripheral vascular disease. Furthermore, the question stem asks about neuropathy, not the condition of the patient's microvasculature.  Answer 5: Sertraline is an antidepressant and is not known to play a role in the formation of diabetic foot ulcers.
  • 74.  A 57-year-old with a 30-year-history of type I diabetes presents to general medical clinic with a lesion on his foot (Figure A). Although he was advised to wear orthotics by his podiatrist, he decided to keep wearing his dress shoes and reports that he observed this lesion when his sock was stained with blood yesterday morning. His vital signs are temperature of 37 degrees Celsius, heart rate 75/minute, blood pressure 145/90 mmHg, respiratory rate 12/minute, and oxygen saturation 99% on room air. Physical examination reveals a diminished response to the monofilament test. What is the most significant risk factor for development of this condition?  1. Peripheral vascular disease  2. Female sex  3. Duration of diabetes mellitus  4. Smoking  5. Neuropath
  • 75.  PREFERRED RESPONSE ▼ 5  DISCUSSION: The greatest risk factor for diabetic foot ulcers is neuropathy. Other risk factors include peripheral vascular disease, bone abnormalities of the foot, male sex, smoking, long term diabetes, and a history of a previous foot ulcer or amputation.  Diabetic foot is caused by a combination of vascular disease (ischemia) and neuropathy which lead to ulcers and infections, sometimes requiring amputation. Neuropathy is the most important risk factor because patients do not feel pain and as a result, repetitive injuries go unnoticed. Furthermore, neuropathy is also a barrier to healing since patients unable to feel pain are less likely to offload pressure on the lesion. Management of diabetic foot ulcers requires offloading with appropriate footware, debridement, and antibiotic therapy if osteomyelitis or cellulitis is present. Optimal blood glucose control, and at times, revascularization also play a role.
  • 76.  A 58-year-old gentleman comes in to his primary care physician's office complaining of "tingling in my fingers and toes". He states that this has occurred more frequently over the past 3-4 weeks and figured it was about time to see his doctor. On physical examination he is noted to be obese and denies any attempts to exercise. A random blood glucose is found to be 223 mg/dL and his hemoglobin A1c is 9.2. What is the most likely diagnosis?  1. Autonomic neuropathy  2. Compression mononeuropathy  3. Symmetrical distal polyneuropathy  4. Proximal neuropathy  5. Focal neuropathy
  • 77.  PREFERRED RESPONSE ▼ 3  DISCUSSION: This individual most likely has diabetes and is experiencing symmetrical distal polyneuropathy.  Diabetics can often be affected by peripheral neuropathy, which is clinically seen as numbness, paresthesias, and pain. The symptoms of distal or peripheral neuropathy often start bilaterally in the toes and feet and gradually rise up the calves and into the knees - the stocking glove" pattern. It's also common for these individuals to experience a burning sensation, decreased deep tendon reflexes and vibration sense along with numbness and paresthesias.
  • 78.  Incorrect Answers:  Answer 1: Although autonomic neuropathy can be seen in diabetics, its presentation is slightly different as it affects the nerves of internal organs such as the bladder muscles, the cardiovascular system, the digestive tract, and the genital organs.  Answer 2: Compressive mononeuropathy refers to a type of entrapment neuropathy that is caused by peripheral nerve injuries at specific locations where a nerve is mechanically constricted in a fibrous or fibro-osseous tunnel.  Answer 4: Proximal neuropathy can also commonly affect type 2 diabetic patients. It predominantly affects the hips, thighs, buttocks, or legs and typically begins on one side of the body.  Answer 5: The head, upper body, or legs may be affected by focal neuropathy, which appears suddenly and is usually painful. Unlike the other forms of neuropathy, this type of neuropathy usually goes away in a few weeks or months and leaves no lasting damage.
  • 79.
  • 80.  A 63-year-old woman with a history of poorly-controlled diabetes mellitus presents to your office to review labs and her blood glucose record. Her hemoglobin A1C is 9.4%. In addition, her blood glucose records demonstrate poor control with numerous spikes and lows throughout the day. Of note, it takes > 1 hr after meals for her blood glucose to rise. She is also complaining of a decreased appetite, nausea, reflux, and early satiety. What is the most appropriate treatment for her current symptoms?  1. Increasing the dose of her short acting insulin  2. Loperamide  3. Metoclopramide  4. Ondansetron  5. Gabapentin
  • 81.  PREFERRED RESPONSE ▼ 3  DISCUSSION: This patient has signs and symptoms of diabetic gastroparesis. The appropriate therapy is a prokinetic such as metoclopramide.  Diabetic gastroparesis is caused by autonomic neuropathy, which may occur with poorly controlled type 1 or type 2 diabetes mellitus. Common symptoms include nausea, vomiting, abdominal pain, early satiety, and poor glucose control. Diagnosis is made on the basis of a gastric emptying scan. Treatment includes a low fiber and low residue diet as well as prokinetic like metoclopramide. Metoclopramide is a dopaminergic receptor antagonist, and a mixed 5-HT3 receptor agonist/antagonist ans has both anti-emetic and prokinetic properties. Severe case of gastroparesis may require a jejunostomy tube.
  • 82.  Incorrect Answers:  Answer 1: Increasing this patient's short acting glucose would not solve her GI symptoms and may precipitate hypoglycemic episodes due to her delayed gastric emptying.  Answer 2: Loperamide would decrease the activity of the myenteric plexus likely make her GI symptoms worse.  Answer 4: Odansetron may help this patient's nausea but will not improve gastric emptying.  Answer 5: Gabapentin is sometimes helpful for relief of symptoms of peripheral neuropathy, but is not regularly used for gastroparesis.
  • 83.
  • 84. ) A 55-year-old male with a medical history significant for type II diabetes mellitus, hypertension, and benign prostatic hypertrophy (BPH) presents to your office with worsening urinary retention. He was started on tamsulosin for his BPH one year ago, and had immediate improvement of his urinary symptoms. However, his urinary symptoms returned and began to worsen after he was started on several new medications at an annual checkup three months ago. The medications started at that time included hydrochlorothiazide for his hypertension, and a combination of amytriptyline and gabapentin for severe diabetic neuropathy. What is the likely culprit of this patient's worsening urinary retention? Topic Review Topic 1. Hydrochlorothiazide 2. Amitriptyline 3. Gabapentin 4. Tachyphylaxis of tamsulosin 5. Progression of his BPH
  • 85.  PREFERRED RESPONSE ▼ 2  DISCUSSION: The anticholinergic effects of the tricyclic (TCA) amitriptyline is the most likely cause of this patient's worsening urinary retention issues.  For medical treatment of diabetic neuropathic pain, first line therapy is generally a TCA, such as amitryptyline (see illustration A for an algorithm in treating diabetic neuropathic pain). If these are contraindicated or do not work alone for the patient, anticonvulsants and/or serotonin-norepinephrine reuptake inhibitors (SNRIs) can be used.  As Lindsay et al. note, tricyclics should not generally be used in patients over the age of 60 years. They are also relatively contraindicated in patients with urinary retention issues/BPH (due to anticholingeric effects), glaucoma, impaired liver function, thyroid disease, and certain cardiac conditions (e.g. arrythmias and orthostatic hypertension).
  • 86.  Incorrect answers:  Answer 1: The major side effects of hydrochlorothiazide are hypokalemia, hyponatremia, hyperglycemia, and hyperlipidemia.  Answer 3: The major side effects of gabapentin are somnolence, dizziness and ataxia.  Answer 4: There is no evidence of tachyphylaxis to tamsulosin.  Answer 5: Although progression of BPH would worsen urinary retention issues, it would not cause these problems in a short period of time.
  • 87.
  • 88. ) A 52-year-old man comes to your clinic for his annual physical exam. He is overweight (BMI=31) and is currently on captopril for hypertension. He reports increased urinary frequency and thirst but is otherwise feeling generally well. He does not exercise and describes his diet as consisting mainly of fast or frozen food. Which of the following values would be sufficient to diagnosis this patient with diabetes? Topic Review Topic 1. Random blood glucose of 185 mg/dL 2. 2 separate fasting glucoses of 130 and 120 mg/dL 3. 2 hour postprandial glucose (glucose tolerance test) of 180 mg/dL 4. Hemoglobin A1c (HbA1c) of 6.8% 5. The presence of diabetic symptoms only
  • 89.  PREFERRED RESPONSE ▼ 4  DISCUSSION: This patient most likely has type II diabetes mellitus (DM) as a rseult of his high BMI, unhealthy lifestyle, and increased urinary frequency and thirst. DM can be diagnosed by a single hemoglobin A1c (HbA1C) of >6.5%.  Type 2 diabetes results from insulin insensitivity in peripheral organs, which results in an increased insulin requirement. Eventually, the pancreas cannot produce enough insulin. The condition has a gradual onset and is typically seen in older individuals, athough it is increasingly found in children with obesity. The HbA1c correlates with the mean glucose from the previous 8-12 weeks (approximate lifespan of a red blood cell). The currently recommended goal for management is < 7%.
  • 90.  Incorrect Answers:  Answer 1: DM is diagnosed with a random blood glucose of at > 200mg/dL AND diabetic symptoms.  Answer 2: DM is diagnosed with 2 separate fasting glucoses >126 mg/dL.  Answer 3: DM is diagnosed with a 2 hour postprandial glucose (glucose tolerance test) > 200 mg/dL.  Answer 5: DM diagnosis requires evidence of high glucose in some form, not simply diabetic symptoms.
  • 91. HYPEROSMOLAR HYPERGLYCEMIC STATE (HHS)/HYPEROSMOLAR NONKETOTIC COMA(HONK) A. Extremely high glucose with profound dehydration b. Occurs in patients with DM type II with lengthy infections, stress, or illness; insulin production is sufficient to prevent DKA by suppressing lipolysis and ketogenesis
  • 92. HHS/HONK SYMPTOMS:  polyuria,  polydipsia,  dehydration,  mental status changes(convulsion,coma)  seizures and  stroke can occur in severe cases
  • 93. HHS/ HONK DIAGNOSIS& TREATMENT  glucose >800 mg/dL (frequently .1,000), no acidosis and extremely high serum osmolality.  A high BUN(prerenal azotemia) and mild metabolic acidosis(bicarbonate~20mEq/L) is also seen without ketosis. Treatment  IV fluids,  insulin,  correction of electrolyte abnormalities;  treat underlying disorder
  • 94. An 81-year-old man with a long history of type II diabetes presents with altered mental status. He is accompanied with his wife who reports that he has become confused over the past twelve hours and has been complaining of thirst. She also notes that he has hardly been urinating. Initial assessment in the ER reveals blood pressure of 90/62 and heart rate of 124 but no fever. The patient is lethargic but has no other pertinent findings on his physical examination. Initial labs reveal a serum glucose of 720 mg/dL. What would be an additional cardinal feature of this clinical syndrome? 1. Ketosis 2. Severe non-anion gap metabolic acidosis 3. Severe anion gap metabolic acidosis 4. Serum osmolarity > 320 mOsm/L 5. Mortality rate < 1%
  • 95.  PREFERRED RESPONSE ▼ 4  This type II diabetic presents with signs and symptoms of hyperglycemic hyperosmolar nonketotic coma. A feature of this condition is serum osmolarity > 320 mOsm/L.  Several different names have been used to describe this state including hyperosmolar hyperglycemic nonketotic syndrome (HHNS), hyperosmolar nonketotic coma (HHNC), and hyperosmolar nonacidotic diabetes. The clinical syndrome refers to a state of severe hyperglycemia, hyperosmolarity, and dehydration, most commonly affecting an elderly type II diabetic. In contrast, type I diabetics commonly present with diabetic ketoacidosis. Nevertheless, pathogenesis is similar. In HHNS, low insulin levels cause hyperglycemia resulting in osmotic diuresis which leads to dehydration. However, ketogenesis is minimal because a small amount of endogenous insulin is present, thus preventing significant acidosis. Diagnostic criteria include hyperglycemia with serum glucose greater than 600 mg/dL and hyperosmolarity with serum osmolarity > 320 mOsm/L, along with no acidosis. Treatment includes fluid and electrolyte replacement and insulin.
  • 96. Incorrect Answers: Answer 1: Ketosis is a laboratory finding in DKA not HHNS. Answers 2 and 3: Typically, HHNS presents without metabolic acidosis. In a minority of cases, it may present with a mild gap lactic acidosis. However, this is not a cardinal symptom. In contrast, in DKA, there is usually a profound anion gap metabolic acidosis from ketone production. Answer 5: Mortality rate is 5-10% in DKA but 10-20% in HHNS. Thus, mortality rate < 1% is not a feature of either of these illnesses.
  • 97.
  • 98.  A 61-year-old female with congestive heart failure and type 2 diabetes is brought to the emergency room by her husband because of an altered mental status. He states he normally helps her be compliant with her medications, but he had been away for several days. On physical exam, her temperature is 37.2 C, BP 85/55, and HR 130. Serum glucose is 500 mg/dL. Which of the following is the first step in the management of this patient? Topic Review Topic  1. IV ½ NS  2. IV NS  3. IV D5W  4. Subcutaneous insulin injection  5. IV insulin
  • 99.  PREFERRED RESPONSE ▼ 2  DISCUSSION: The patient in this vignette is most likely suffering from non-ketotic hyperosmolar syndrome (NKHS). The first step in the management of this condition is the administration of IV NS.  NKHS is a complication of type 2 diabetes. The concentration of insulin required to prevent ketoacidosis is lower than the concentration required to adequately control glucose levels. Thus, the blood glucose may be very high, but ketoacidosis will be absent (unlike in type 1 DM). NKHS is seen most commonly in older patients with concurrent cardiac and renal pathology. This conditon presents with symptoms and signs of shock. There is a high mortality with NKHS, approximately 50%.
  • 101. PRESENTATION  faintness,  weakness,  diaphoresis, and  palpitations because of responsive excess secretion of epinephrine (attempt to mobilize glycogen);  headache,  confusion,  mental status changes, and  decreased consciousness because of inadequate supply of glucose to the brain
  • 102.
  • 103.  A 74-year-old male is brought to the emergency department by his son during an episode of dizziness, diaphoresis, and nervousness. The patient's past medical history includes diabetes mellitus type 2, hypertension, and end-stage renal disease (ESRD) for which the patient is on dialysis. Medications include glyburide and valsartan. Serum glucose is 33 mg/dL, and subsequent testing shows elevated serum sulfonylurea levels. Symptoms improve upon intravenous administration of 50% dextrose but blood glucose remains below 80 mg/dL for 24 hours. Which of the following tests is NOT indicated in the work-up of this patient's hypoglycemia? Topic Review Topic  1. Serum proinsulin level  2. Urine 5-hydroxyindolacetic acid (5-HIAA) level  3. Serum insulin-like growth factor II (IGF-II) level  4. Serum C-peptide level  5. Serum insulin level
  • 104.  PREFERRED RESPONSE ▼ 2  DISCUSSION: The patient presents with dizziness, diaphoresis, and nervousness and a blood sugar of 33 mg/dL consistent with symptomatic hypoglycemia.  Hypoglycemia is associated with multiple differential diagnoses. In this patient with chronic kidney disease, the use of glyburide, a sulfonylurea cleared by the kidneys, is the agent most likely to be responsible for the patient's hypoglycemic episode. Serum insulin, pro-insulin, c-peptide, and IGF-II levels can help elucidate alternative diagnoses. Urine 5-HIAA levels are elevated in carcinoid syndrome and do not contribute to the work-up of hypoglycemia.  Havas discusses hypoglycemia in the management of type I diabetes. Fatigue, mental slowing, confusion, irritability, weakness, pallor and twitching are all symptoms that may indicate hypoglycemia. Monitoring blood glucose frequently, maintaining a prudent diet and regular exercise can help minimize the risk of significant hypoglycemia.
  • 105.  Incorrect Answers:  Answers 1 and 4: Serum C-peptide levels and proinsulin levels are decreased in the setting of exogenous insulin use.  Answer 3: Elevated IGF-II levels can be found in the setting of acromegaly, a potential underlying cause of hypoglycemia.  Answer 5: Serum insulin levels help measure if the body's response to hypoglycemia is appropriate. When blood glucose levels are below 60 mg/dL, serum insulin should be almost entirely suppresed.
  • 106.  A 52-year-old woman comes to your clinic for her annual physical exam. She is obese, does not exercise, and regularly eats fried foods. A random blood glucose is 249 mg/dL. Her hemoglobin A1C is 9.5. Which of the following treatments would be weight neutral or cause weight loss in this patient? Topic Review Topic  1. Glargine  2. Glyburide  3. Pioglitazone  4. Metformin  5. Glipizide
  • 107.  4  DISCUSSION: The patient in this vignette most likely has type II diabetes. Of the given treatments, only metformin is weight neutral in the majority of cases (i.e. it does not cause significant weight gain/loss).  Metformin is a first-line treatment for type II DM in most patients. Although the exact mechanism is unknown, it appears to decrease gluconeogenesis and increase insulin sensitivity. There is no risk of hypoglycemia or weight gain (though some patients even lose weight). The most high yield side effect involves lactic acidosis, particularly in patients with renal insufficiency.
  • 108.
  • 109.  incorrect Answers:  Answer 1: Glargine, a long-acting insulin, can cause weight gain.  Answer 2: Glyburide, a second generation sulfonylurea, can cause weight gain.  Answer 3: Pioglitazone, a thiazolidinedione, can cause weight gain.  Answer 5: Glipizide, a second generation sulfonylurea, can cause weight gain.